Baccalauréat Polynésie 11 septembre 2014 STI2D--STL spécialité SPCL

Exercice 1 4 points


Nombres complexes

On considère les nombres complexes $Z_{1}$ et $Z_{2}$ : \[Z_{1} = \dfrac{3\sqrt{2}}{1 + \text{i}}\quad \text{et}\quad Z_{2} = \dfrac{4\text{i}}{1 + \text{i}\sqrt{3}}.\]

  1. Ecrire les nombres $Z_{1}$ et $Z_{2}$ sous forme algébrique et trigonométrique.
  2. Placer les points A$_{1}$ et A$_{2}$ d'affixes respectives $Z_{1}$ et $Z_{2}$ dans le repère donné en annexe.
  3. Calculer sous forme algébrique le produit $Z_{1} \times Z_{2}$ et donner sa forme trigonométrique.
  4. En déduire les valeurs exactes de $\cos \dfrac{\pi}{12}$ et $\sin \dfrac{\pi}{12}$.

Bac STI2D Polynesie Sept 2014 Ex1 


Correction de l'exercice 1 (4 points)


Nombres complexes

On considère les nombres complexes $Z_{1}$ et $Z_{2}$ : \[Z_{1} = \dfrac{3\sqrt{2}}{1 + \text{i}}\quad \text{et}\quad Z_{2} = \dfrac{4\text{i}}{1 + \text{i}\sqrt{3}}.\]

  1. Ecrire les nombres $Z_{1}$ et $Z_{2}$ sous forme algébrique et trigonométrique.
    • Forme algébrique de $Z_1$
    • $$\begin{array}{ll} Z_{1} &= \dfrac{3\sqrt{2}}{1 + \text{i}} \\ & = \dfrac{3\sqrt{2}(1 - \text{i})}{(1 + \text{i})(1 - \text{i})}\\ & = \dfrac{3\sqrt{2} - 3\sqrt{2}\text{i}}{ 1^2+1^2 }\\ & = \dfrac{3\sqrt{2} - 3\sqrt{2}\text{i}}{2 }\\ \end{array}$$
      $ Z_1= \dfrac{3\sqrt{2} - 3\sqrt{2}\text{i}}{2 }$.
    • Forme trigonométrique de $Z_1$
    • Forme trigonométrique de $ Z_1= \dfrac{3\sqrt{2} - 3\sqrt{2}\text{i}}{2 }=\dfrac{3\sqrt{2}}{2} -\text{i} \dfrac{3\sqrt{2}}{2}$:
      Module : $|Z_1|=\sqrt{a^2+b^2}=\sqrt{\left(\dfrac{3\sqrt{2}}{2}\right)^2+\left(\dfrac{3\sqrt{2}}{2}\right)^2}=\sqrt{\dfrac{18}{4}+\dfrac{18}{4}}= \sqrt{9}=3$
      Argument: $$\left\{ \begin{array}{l } \cos(\theta)=\dfrac{a}{r}= \dfrac{\dfrac{3\sqrt{2}}{2}}{3}= \dfrac{\sqrt 2}{2}\\ \sin(\theta)=\dfrac{b}{r}= \dfrac{\dfrac{-3\sqrt{2}}{2}}{3}= -\dfrac{\sqrt 2}{2} \end{array} \right.$$ Ainsi $\theta=-\dfrac{\pi}{4}$ convient; on a donc: $$Z_1=[3;-\dfrac{\pi}{4}] \text{ ou } Z_1=3\left [\cos\left (-\dfrac{\pi}{4}\right )+i\sin\left (-\dfrac{\pi}{4}\right )\right ]$$
      La forme exponentielle de $Z_{1}$ est $Z_1= 3e^{-i\frac{\pi}{4}}$
      La forme trigonométrique de $Z_1$ est $Z_1=3\left [\cos\left (-\dfrac{\pi}{4}\right )+i\sin\left (-\dfrac{\pi}{4}\right )\right ]$
    • Forme algébrique de $Z_2$
    • $$\begin{array}{ll} Z_{2} &= Z_{2} = \dfrac{4\text{i}}{1 + \text{i}\sqrt{3}} \\ & = \dfrac{4\text{i}(1 - \text{i}\sqrt{3})}{(1 + \text{i}\sqrt{3})(1 - \text{i}\sqrt{3})}\\ & = \dfrac{4\text{i} + 4 \sqrt{3}}{ 1^2+\sqrt 3^2 }\\ & = \dfrac{4 \sqrt{3} 4\text{i}}{4 }\\\end{array}$$
      $ Z_2= \sqrt{3} +\text{i}$.
    • Forme trigonométrique de $Z_2$
    • Forme trigonométrique de $Z_2=\sqrt{3} +\text{i}$:
      Module : $|Z_2|=\sqrt{a^2+b^2}=\sqrt{\sqrt{3}^2+1^2}=\sqrt{4}=2$
      Argument: $$\left\{ \begin{array}{l } \cos(\theta)=\dfrac{a}{r}= \dfrac{\sqrt{3}}{2}\\ \sin(\theta)=\dfrac{b}{r}= \dfrac{1}{2} \end{array} \right.$$ Ainsi $\theta= \dfrac{\pi}{6}$ convient; on a donc: $$Z_2=[2; \dfrac{\pi}{6}] \text{ ou } Z_2=2\left [\cos\left (\dfrac{\pi}{6}\right )+i\sin\left (\dfrac{\pi}{6}\right )\right ]$$
      La forme exponentielle de $Z_{2}$ est $Z_2= 2e^{i\frac{\pi}{6}}$
      La forme trigonométrique de $Z_2$ est $Z_2=2\left [\cos\left (\dfrac{\pi}{6}\right )+i\sin\left (\dfrac{\pi}{6}\right )\right ]$
  2. Placer les points A$_{1}$ et A$_{2}$ d'affixes respectives $Z_{1}$ et $Z_{2}$ dans le repère donné en annexe.
  3. Calculer sous forme algébrique le produit $Z_{1} \times Z_{2}$ et donner sa forme trigonométrique.
    • Forme algébrique de $Z_1\times Z_2$
    • $$\begin{array}{ll} Z_{1}\times Z_2 &= \dfrac{3\sqrt{2} - 3\sqrt{2}\text{i}}{2 }\times ( \sqrt{3} +\text{i}) \\ & = \dfrac{3\sqrt{6}}{2} +\text{i} \dfrac{3\sqrt{2}}{2}- \text{i} \dfrac{3\sqrt{6}}{2} +\dfrac{3\sqrt{2}}{2}\\ & = \dfrac{3\sqrt{2} - 3\sqrt{2}\text{i}}{ 1^2+1^2 }\\ & = \dfrac{3(\sqrt{6}+\sqrt{2})}{2} + \text{i} \dfrac{3(\sqrt{2}-\sqrt{6})}{2}\\\end{array}$$
      $ Z_1\times Z_2 = \dfrac{3(\sqrt{6}+\sqrt{2})}{2} + \text{i} \dfrac{3(\sqrt{2}-\sqrt{6})}{2}$.
    • Forme trigonométrique de $Z_1\times Z_2$
    • $$\begin{array}{ll} Z_{1}\times Z_2 &= 3e^{-i\frac{\pi}{4}} \times 2e^{i\frac{\pi}{6}} \\ & = 6 e^{i\left(-\frac{\pi}{4} + \frac{\pi}{6}\right)} \\ & = 6 e^{-i\frac{\pi}{12} } \\\\\end{array}$$
      La forme trigonométrique de $ Z_1\times Z_2$ est $ Z_1\times Z_2 = 6 e^{-i\frac{\pi}{12} }$
  4. En déduire les valeurs exactes de $\cos \dfrac{\pi}{12}$ et $\sin \dfrac{\pi}{12}$.
  5. On identifie les formes algébrique et trigonométrique de $ Z_1\times Z_2$: $$ Z_1\times Z_2 = 6 e^{-i\frac{\pi}{12} }=6\left [\cos\left (\dfrac{-\pi}{12}\right )+i\sin\left (\dfrac{-\pi}{12}\right )\right ]$$ $$\begin{array}{ll } Z_1\times Z_2&= \dfrac{3(\sqrt{6}+\sqrt{2})}{2} + \text{i} \dfrac{3(\sqrt{2}-\sqrt{6})}{2} \\ &= 6\left [\cos\left (\dfrac{-\pi}{12}\right )+i\sin\left (\dfrac{-\pi}{12}\right )\right ] \end{array}$$ En identifiant les parties réelles et les parties imaginaires on obtient : $$\left\{ \begin{array}{l } Re(Z_1\times Z_2 )=6\cos(\dfrac{-\pi}{12})= 6\cos(\dfrac{\pi}{12})=\dfrac{3(\sqrt{6}+\sqrt{2})}{2} \\ Im(Z_1\times Z_2 )=6\sin(-\dfrac{\pi}{12})=-6\sin(\dfrac{\pi}{12})= \dfrac{3(\sqrt{2}-\sqrt{6})}{2} \end{array} \right.$$
    $$\left\{ \begin{array}{l } \cos(\dfrac{ \pi}{12})=\dfrac{ \sqrt{6}+\sqrt{2} }{4} \\ \sin(\dfrac{\pi}{12})= \dfrac{ \sqrt{6}-\sqrt{2} }{4} \end{array} \right.$$

Exercice 2 6 points


Suites


Une entreprise informatique a réalisé en 2013 un bénéfice de 22000 €. La direction de cette entreprise se fixe pour objectif une hausse annuelle de son bénéfice de 4,5$\,\%$. Pour tout entier naturel $n$, on note $b_{n}$ le bénéfice prévu pour l'année $2013 + n$, on a donc $b_{0} = 22000$.


Partie A

  1. Calculer les bénéfices $b_{1}$ et $b_{2}$ espérés pour 2014 et 2015.
  2. Montrer que $\left(b_{n}\right)$ est une suite géométrique dont on précisera les éléments caractéristiques.
  3. Exprimer alors $b_{n}$ en fonction de $n$.
Partie B

On considère l'algorithme ci-dessous :

$$\begin{array}{|l|}\hline N \text{ prend la valeur } 0\\ B \text{ prend la valeur }22000 \\ \text{ Tant que } B \leqslant 40000 \\ \hspace{0,5cm}N \text{ prend la valeur } N + 1\\ \hspace{0,5cm}B \text{ prend la valeur }1,045 * B\\ \text{ Fin Tant que } \\ A \text{ prend la valeur } N + 2013\\ \text{ Afficher } A\\ \hline \end{array}$$

  1. Expliquer à quoi correspondent les variables N et B.
  2. Exécuter cet algorithme et donner le dernier résultat affiché.
  3. Expliquer à quoi correspond cette valeur.
  4. La direction souhaite savoir à partir de quelle année le bénéfice de l'entreprise sera supérieur à 40000 €.
    1. Résoudre dans $\mathbb R$ l'inéquation suivante: \[22000 \times 1,045^x > 40000.\]
    2. Quel lien existe-t-il entre le résultat de la question 2. de la partie B et l'ensemble des solutions de l'inéquation précédente ?

 


Correction de l'exercice 2 (6 points)


Suites


Une entreprise informatique a réalisé en 2013 un bénéfice de 22000 €. La direction de cette entreprise se fixe pour objectif une hausse annuelle de son bénéfice de 4,5$\,\%$. Pour tout entier naturel $n$, on note $b_{n}$ le bénéfice prévu pour l'année $2013 + n$, on a donc $b_{0} = 22000$.


Partie A

  1. Calculer les bénéfices $b_{1}$ et $b_{2}$ espérés pour 2014 et 2015.
  2. $b_1=22000\times \left(1+\dfrac{4,5}{100}\right)=22990$ $b_2=22990\times \left(1+\dfrac{4,5}{100}\right)= 24024,55$
    Les bénéfices espérés pour 2014 et 2015 sont respectivement de 22990 € et 24024,55 €.
  3. Montrer que $\left(b_{n}\right)$ est une suite géométrique dont on précisera les éléments caractéristiques.
  4. $b_0=22000$ et, pour tout entier naturel $n, b_{n+1}=1,045\times b_n$
    donc $(b_n)$ est une suite géométrique de raison $1,045$ et de premier terme $22000$.
  5. Exprimer alors $b_{n}$ en fonction de $n$.
  6. $(b_n)$ est une suite géométrique de raison $1,045$ et de premier terme $22000$, $b-n=q^n\times b-0$
    alors pour tout entier $n, b_n=22000\times 1,045^n$
Partie B

On considère l'algorithme ci-dessous :$$\begin{array}{|l|}\hline N \text{ prend la valeur } 0\\ B \text{ prend la valeur }22000 \\ \text{ Tant que } B \leqslant 40000 \\ \hspace{0,5cm}N \text{ prend la valeur } N + 1\\ \hspace{0,5cm}B \text{ prend la valeur }1,045 * B\\ \text{ Fin Tant que } \\ A \text{ prend la valeur } N + 2013\\ \text{ Afficher } A\\ \hline \end{array}$$

  1. Expliquer à quoi correspondent les variables N et B.
  2. $N$ est le nombre d'années écoulées depuis 2013 et $B$ est le montant en euros du bénéfice prévu pour l'année $2013+N.$
  3. Exécuter cet algorithme et donner le dernier résultat affiché.
  4. Le résultat affiché est $2027$
  5. Expliquer à quoi correspond cette valeur.
  6. C'est à partir de 2027 que le bénéfice sera supérieur à 40000 €.
    La direction souhaite savoir à partir de quelle année le bénéfice de l'entreprise sera supérieur à 40000 €.
    1. Résoudre dans $\mathbb R$ l'inéquation suivante: \[22000 \times 1,045^x > 40000.\]
    2. Pour tout réel $x$, on a $$\begin{array}{ll} 22000 \times 1,045^x > 40000 &\iff 1,045^x > \dfrac{40000}{22 000}\\ &\iff e^{x \ln(1,045)} > \dfrac{20}{11}\\ &\iff \ln\left(e^{x \ln(1,045)} \right)> \ln\left(\dfrac{20}{11}\right)\\ &\iff x \ln(1,045) > \ln\left(\dfrac{20}{11}\right)\\ &\iff x > \dfrac{\ln\left(\dfrac{20}{11}\right)}{\ln(1,045)}\\ \end{array}$$
      L’ensemble des solutions de l’inéquation est l’intervalle $]\dfrac{\ln20−\ln11}{\ln1,045};+\infty[$.
    3. Quel lien existe-t-il entre le résultat de la question 2. de la partie B et l'ensemble des solutions de l'inéquation précédente ?
    4. L'algorithme de la question 2 permet de déterminer le rang $N$ de l'année à partir de laquelle le bénéfice de l'entreprise sera supérieur à $40000$ €. $N$ est le plus petit entier solution de l'inéquation $22000 \times 1,045^x > 40000$.
      REMARQUE : Comme $\dfrac{\ln20−\ln11}{\ln1,045}\approx 13,582$, le plus petit entier $N$ tel que $N>\dfrac{\ln20−\ln11}{\ln1,045}$ est $N=14$. On retrouve le résultat de la question 2.

 


Exercice 3 6 points


Fonction exponentielle et équation différentielle


Lorsque l'on consomme de l'alcool, le taux d'alcool dans le sang varie en fonction du temps écoulé depuis l'absorption. Ce taux est appelé « alcoolémie » et est mesuré en grammes par litre (g/L). Après l'absorption de trois verres d'alcool, l'alcoolémie d'une personne donnée, en fonction du temps (exprimé en heures), est modélisée par la fonction définie sur $\mathbb R_{+}$ par : \[ f(t) = 2,5t\text{e}^{- t}.\]

Partie A

  1. Donner la valeur de l'alcoolémie de la personne considérée au bout de 2 heures.
  2. Montrer que pour tout réel $t$ de l'intervalle $[0~;~+ \infty[$,  $f'(t) = 2,5(1 - t)\text{e}^{- t}$.
  3. Vérifier que la fonction $f$ est solution de l'équation différentielle : \[(E) :\qquad y' + y = 2,5\text{e}^{- t}.\]
  4. En remarquant que pour tout réel $t$ de l'intervalle $[0~;~+ \infty[$ on a $f(t) = \dfrac{2,5t}{\text{e}^{t}}$, déterminer $\displaystyle\lim_{t \to + \infty}f(t)$ et donner une interprétation géométrique de cette limite.
  5. Déterminer les variations de la fonction $f$ sur l'intervalle $[0~;~+ \infty[$.
  6. Quelle est l'alcoolémie la plus élevée pour la personne considérée ?

Partie B

  1. Sur une feuille de papier millimétré, tracer la courbe représentative de la fonction $f$ sur l'intervalle $[0~;~+ \infty[$. On prendra 2 cm pour unité sur l'axe des abscisses et 10 cm pour unité sur l'axe des ordonnées.
  2. En France, la législation autorise pour un conducteur une alcoolémie maximale de 0,5 g/L. Sachant que la personne a absorbé trois verres d'alcool à 12 h, à partir de quelle heure pourra-t-elle reprendre la route pour effectuer sans s'arrêter un trajet d'une durée d'une heure ? On utilisera la représentation graphique de la fonction $f$.

 


Correction de l'exercice 3 (6 points)


Fonction exponentielle et équation différentielle


Lorsque l'on consomme de l'alcool, le taux d'alcool dans le sang varie en fonction du temps écoulé depuis l'absorption. Ce taux est appelé « alcoolémie » et est mesuré en grammes par litre (g/L). Après l'absorption de trois verres d'alcool, l'alcoolémie d'une personne donnée, en fonction du temps (exprimé en heures), est modélisée par la fonction définie sur $\mathbb R_{+}$ par : \[ f(t) = 2,5t\text{e}^{- t}.\]

Partie A

  1. Donner la valeur de l'alcoolémie de la personne considérée au bout de 2 heures.
  2. $f(2)=2,5\times 2\times e{-2}\approx 0,68$
    Au bout de 2 heures, l'alcoolémie de la personne considérée est d'environ 0,68 g/L.
  3. Montrer que pour tout réel $t$ de l'intervalle $[0~;~+ \infty[$,  $f'(t) = 2,5(1 - t)\text{e}^{- t}$.
  4. $f $ est dérivable comme produit de deux fonctions dérivables : 

    $f=uv$ d'où $f'=u' v+uv' $ avec pour tout réel $x$ :
    $$\left\{ \begin{array}{l} u(t)~ =2,5t \\ v(t)~ =\text{e}^{- t} \end{array}\right.$$ d'où : $$\left\{ \begin{array}{l} u’(t)~ =2,5 \\ v'(t)~ =-\text{e}^{- t} \end{array}\right.$$

    Ainsi :
     $$f’(t)=2,5 \times \text{e}^{- t} +2,5t \times \left( -\text{e}^{- t}\right)$$

    $$f’(t)=2,5(1-t)\text{e}^{-t}$$
    La dérivée de la fonction $f$ est la fonction $f’$ définie pour tout réel $t$ de l'intervalle $[0;+\infty[$ par $f’(t)=2,5(1−t)\text{e}^{-t}$.
  5. Vérifier que la fonction $f$ est solution de l'équation différentielle : \[(E) :\qquad y' + y = 2,5\text{e}^{- t}.\]
  6. $$\begin{array}{ll} f’(t)+f(t)&=2,5(1−t)\text{e}^{-t}+2,5t\text{e}^{-t}\\ &= 2,5 \text{e}^{- t} \end{array}$$
    La fonction f est une solution de l'équation différentielle (E).
  7. En remarquant que pour tout réel $t$ de l'intervalle $[0~;~+ \infty[$ on a $f(t) = \dfrac{2,5t}{\text{e}^{t}}$, déterminer $\displaystyle\lim_{t \to + \infty}f(t)$ et donner une interprétation géométrique de cette limite.
  8. $\displaystyle\lim_{t \to + \infty}\dfrac{\text{e}^{t}}{t}=+\infty$, donc par inverse : $\displaystyle\lim_{t \to + \infty}\dfrac{t}{\text{e}^{t}}=0$, puis $\displaystyle\lim_{t \to + \infty}\dfrac{2,5t}{\text{e}^{t}}=0$
    $\displaystyle\lim_{t \to + \infty}f(t)=0$ par conséquent, la courbe représentative de la fonction $f$ admet pour asymptote l'axe des abscisses au voisinage de $+ \infty$.
  9. Déterminer les variations de la fonction $f$ sur l'intervalle $[0~;~+ \infty[$.
  10. Le sens de variation de $f$ est donné par le signe de la dérivée $f’$; or $f’(t)=2,5(1-t)\text{e}^{-t}$
    Comme la fonction exponentielle est strictement positive sur $\mathbb{R}$ ; le signe de $f′(t)$ ne dépend que de celui de $1-t$. On obtient donc le tableau de variations suivant :
  11. Quelle est l'alcoolémie la plus élevée pour la personne considérée ?
  12. Le maximum de la fonction $f$ est atteint pour $t=1$ et, $$f(1)=2,5\times \text{e}^{-1}\approx 0,92$$
    L'alcoolémie la plus élevée pour la personne considérée est d'environ 0,92 g/L.

Partie B

  1. Sur une feuille de papier millimétré, tracer la courbe représentative de la fonction $f$ sur l'intervalle $[0~;~+ \infty[$. On prendra 2 cm pour unité sur l'axe des abscisses et 10 cm pour unité sur l'axe des ordonnées.
  2. En France, la législation autorise pour un conducteur une alcoolémie maximale de 0,5 g/L. Sachant que la personne a absorbé trois verres d'alcool à 12 h, à partir de quelle heure pourra-t-elle reprendre la route pour effectuer sans s'arrêter un trajet d'une durée d'une heure ? On utilisera la représentation graphique de la fonction $f$.
  3. Avec la précision permise par un graphique tracé à main levée, on constate que la courbe représentative de la fonction $f$ est en dessous de la droite d'équation $y=0,5$ sur un intervalle d'amplitude 1 pour $t>2,5$. Or $f(2,5)\approx 0,513$ et $f(2,55)\approx 0,498$
    Cette personne pourra reprendre la route pour effectuer sans s'arrêter un trajet d'une durée d'une heure à partir de 14 heures trente-cinq minutes.

Exercice 4 4 points


Probabilités


Partie A Loi exponentielle et radioactivité

On modélise la durée de vie $T$ (exprimée en jours) d'un élément radioactif par une variable aléatoire suivant une loi exponentielle de paramètre $\lambda$.
On rappelle que pour tout $t > 0,\: P(T \leqslant t) = \displaystyle\int_{0}^t \lambda\text{e}^{- \lambda x}\:\text{d}x$.
Le Thorium 227 a une demi-vie de 18 jours, ce qui signifie que : \[P(T \geqslant 18) = P(T \leqslant 18) = 0,5.\]

  1. Montrer que pour tout $t > 0,\: P(T \leqslant t) = 1 - \text{e}^{-\lambda t}$.
  2. Calculer la valeur du paramètre $\lambda$ pour le Thorium 227. On donnera le résultat arrondi à $10^{-4}$.
  3. On suppose que $\lambda = 0,04$. Donner alors la durée de vie moyenne d'un atome de Thorium 227.

Partie B Loi normale et usinage

Une entreprise fabrique en grande quantité des pièces tubulaires destinées à l'industrie aérospatiale. Le diamètre (exprimé en centimètres) d'une de ces pièces est modélisé par une variable aléatoire $X$ suivant la loi normale d'espérance $3,65$ et d'écart type $0,004$.
Les résultats seront donnés à $10^{-3}$ près .


  1. Une pièce est décrétée conforme lorsque son diamètre en centimètres est compris entre $3,645$ et $3,655$. Calculer la probabilité qu'une pièce tubulaire de la production soit décrétée conforme.
  2. Dans le cadre d'un fonctionnement correct de la chaîne de production, on admet que la proportion $p$ de pièces conformes est 79$\,\%$. On rappelle que l'intervalle de fluctuation asymptotique à 95$\,\%$ de la fréquence de pièces conformes sur un échantillon de taille $n$ est \[I = \left[p - 1,96\sqrt{\dfrac{p(1 - p)}{n}}~;~p + 1,96\sqrt{\dfrac{p(1 - p)}{n}}\right].\] On contrôle régulièrement la chaîne de production en prélevant des échantillons de $100$ pièces. Lors d'un contrôle, on trouve 25 pièces défectueuses. Le responsable qualité doit-il prendre la décision d'effectuer des réglages sur la chaîne de production ? Justifier la réponse.

 


Exercice 4 : 4 points


Probabilités


Partie A Loi exponentielle et radioactivité

On modélise la durée de vie $T$ (exprimée en jours) d'un élément radioactif par une variable aléatoire suivant une loi exponentielle de paramètre $\lambda$.
On rappelle que pour tout $t > 0,\: P(T \leqslant t) = \displaystyle\int_{0}^t \lambda\text{e}^{- \lambda x}\:\text{d}x$.
Le Thorium 227 a une demi-vie de 18 jours, ce qui signifie que : \[P(T \geqslant 18) = P(T \leqslant 18) = 0,5.\]

  1. Montrer que pour tout $t > 0,\: P(T \leqslant t) = 1 - \text{e}^{-\lambda t}$.
  2. $$\begin{array}{ll} P(T \leqslant t)& = \displaystyle\int_{0}^t \lambda\text{e}^{- \lambda x}\:\text{d}x\\ &=\left[ -\text{e}^{- \lambda x}\right]_0^t\\ &= -\text{e}^{- \lambda t}-(-\text{e}^{0})\\ & 1 -\text{e}^{- \lambda t}\\ \end{array}$$
    pour tout $t > 0,\: P(T \leqslant t) = 1 - \text{e}^{-\lambda t}$.
  3. Calculer la valeur du paramètre $\lambda$ pour le Thorium 227. On donnera le résultat arrondi à $10^{-4}$.
  4. $$\begin{array}{ll} 1 -\text{e}^{- 18\lambda } =0,5&\iff \text{e}^{- 18\lambda } =0,5 \\ &\iff \ln\left(\text{e}^{- 18\lambda } \right)=\ln (0,5)\\ &\iff - 18\lambda= \ln (0,5)\\ &\iff \lambda = -\dfrac{\ln (0,5)}{18}\approx 0,0385 \end{array}$$
    La durée de vie du Thorium 227 suit la loi exponentielle de paramètre $\lambda =0,0385$.
  5. On suppose que $\lambda = 0,04$. Donner alors la durée de vie moyenne d'un atome de Thorium 227.
  6. L'espérance d'une variable aléatoire suivant une loi exponentielle de paramètre $\lambda$ est égale à $\dfrac{1}{\lambda}$.
    La durée de vie moyenne d'un atome de Thorium 227 est de $\dfrac{1}{0,04} =25$ jours.

Partie B Loi normale et usinage

Une entreprise fabrique en grande quantité des pièces tubulaires destinées à l'industrie aérospatiale. Le diamètre (exprimé en centimètres) d'une de ces pièces est modélisé par une variable aléatoire $X$ suivant la loi normale d'espérance $3,65$ et d'écart type $0,004$.
Les résultats seront donnés à $10^{-3}$ près .


  1. Une pièce est décrétée conforme lorsque son diamètre en centimètres est compris entre $3,645$ et $3,655$. Calculer la probabilité qu'une pièce tubulaire de la production soit décrétée conforme.
  2. 2ND DISTR 2NORMALFRép( \1 , \2,\3,\4)EXE
    Avec une calculatrice de type TI

    $$NormalFR\text{é}p(\1,\2,\3,\4) \approx \5$$

    $$P(\1 \leq \6 \leq \2)\approx \5 \text{ à } 10^{-\7} \text{ près.}$$

     

    La probabilité qu'une pièce tubulaire de la production soit décrétée conforme est $0,789$ (arrondie au millième près).
  3. Dans le cadre d'un fonctionnement correct de la chaîne de production, on admet que la proportion $p$ de pièces conformes est 79$\,\%$. On rappelle que l'intervalle de fluctuation asymptotique à 95$\,\%$ de la fréquence de pièces conformes sur un échantillon de taille $n$ est \[I = \left[p - 1,96\sqrt{\dfrac{p(1 - p)}{n}}~;~p + 1,96\sqrt{\dfrac{p(1 - p)}{n}}\right].\] On contrôle régulièrement la chaîne de production en prélevant des échantillons de $100$ pièces. Lors d'un contrôle, on trouve 25 pièces défectueuses. Le responsable qualité doit-il prendre la décision d'effectuer des réglages sur la chaîne de production ? Justifier la réponse.
  4. La proportion $p$ est égale à  $\1$. La taille  $n$  de l'échantillon considéré est égale à  $\2.$
    Comme  $ n =\2$ ,   $n \times p  $=\3  et $n\times (1-p)=\4,$ les conditions d'utilisation d'un intervalle de fluctuation asymptotique sont réunies.

    En effet on a bien : $$n \geq 30\;;\; n \times p \geq 5 \text{ et } n\times (1-p) \geq 5$$


    L'intervalle de fluctuation asymptotique au seuil de  $95\% $  est : $$I_{\2} = \left[\1 - 1,96\sqrt{\dfrac{\1\times \5}{\2}}~;~\1 + 1,96\sqrt{\dfrac{\1\times \5}{\2}} \right]$$ 

    Soit en prenant des valeurs approchées à $10^{-3 }$ près des bornes de l'intervalle, l'intervalle de fluctuation asymptotique au seuil de 95  %  de la fréquence de pièces conformes sur un échantillon de taille 100 est $I_{100}=[0,710;0,870]$.
    La fréquence observée de pièces conformes dans l'échantillon est $f=\dfrac{100-25}{100}≈0,75$
    La fréquence observée appartient à l'intervalle de fluctuation asymptotique au seuil de 95  %  . Il n'est pas nécessaire d'effectuer des réglages sur la chaîne de production.
  • Vues: 12331

Baccalauréat STI 2D/STL spécialité SPCL Métropole 11 septembre 2014

Exercice 1 5 points


QCM


Cet exercice est un questionnaire à choix multiples. Pour chacune des questions suivantes, une seule des quatre réponses proposées est exacte. Aucune justification n'est demandée.
Une bonne réponse rapporte un point. Une mauvaise réponse, plusieurs réponses ou l'absence de réponse à une question ne rapportent ni n'enlèvent de point.


Indiquer sur la copie le numéro de la question et la réponse correspondante.


  1. La forme exponentielle du nombre complexe $z_{1} = \sqrt{6} + \text{i}\sqrt{6}$ est :
    1. $z_{1} = 2\sqrt{3}\text{e}^{\text{i}\frac{\pi}{4}}$
    2. $z_{1} = 2\sqrt{6}\text{e}^{-\text{i}\frac{\pi}{4}}$
    3. $z_{1} = 6\text{e}^{\text{i}\frac{\pi}{4}}$
    4. $z_{1} = \sqrt{2}\text{e}^{\text{i}\frac{7\pi}{4}}$
  2. On considère les nombres complexes $z_{1} = \sqrt{6} + \text{i}\sqrt{6}$ et $z_{2} = - \sqrt{6} + \text{i}\sqrt{6}$. Le nombre complexe $z_{2}$ est égal à :
    1. $\overline{z_{1}}$
    2. $- z_{1}$
    3. $- \overline{z_{1}}$
    4. $\text{i} + z_{1}$
  3. La fonction $f$ est définie sur l'intervalle $]0 ; +\infty[$ par $f(x) = \dfrac{1}{x}$. Sa courbe représentative est donnée ci-dessous :

    Le domaine du plan défini comme l'ensemble des points $M$ de coordonnées $(x ; y)$ qui vérifient $1 \leqslant x \leqslant 2$ et $\dfrac{1}{x} \leqslant y \leqslant 1$ a pour aire (exprimée en unité d'aire) :
    1. $\ln 2$
    2. $\dfrac{1}{2}$
    3. $1 - \ln 2$
    4. $1 - \text{e}^2$
  4. La tangente au point d'abscisse $\dfrac{1}{2}$ à la courbe représentative de la fonction $f$, définie sur l'intervalle $]0 ; + \infty[$ par $f(x) = \dfrac{1}{x}$, a pour équation :
    1. $y = - 4x + 4$
    2. $y = \phantom{-}4x + 4$
    3. $y = - 4x - 4$
    4. $y = \phantom{-}4x - 4$

 


Correction de l'exercice 1 (5 points)


QCM

Cet exercice est un questionnaire à choix multiples. Pour chacune des questions suivantes, une seule des quatre réponses proposées est exacte. Aucune justification n'est demandée.
Une bonne réponse rapporte un point. Une mauvaise réponse, plusieurs réponses ou l'absence de réponse à une question ne rapportent ni n'enlèvent de point.


Indiquer sur la copie le numéro de la question et la réponse correspondante.


  1. La forme exponentielle du nombre complexe $z_{1} = \sqrt{6} + \text{i}\sqrt{6}$ est :
    1. $z_{1} = 2\sqrt{3}\text{e}^{\text{i}\frac{\pi}{4}}$
    2. $z_{1} = 2\sqrt{6}\text{e}^{-\text{i}\frac{\pi}{4}}$
    3. $z_{1} = 6\text{e}^{\text{i}\frac{\pi}{4}}$
    4. $z_{1} = \sqrt{2}\text{e}^{\text{i}\frac{7\pi}{4}}$
  2. Forme trigonométrique de $z_1=\sqrt{6} + \text{i}\sqrt{6}$:
    Module : $|z_1|=\sqrt{a^2+b^2}=\sqrt{\sqrt{6}^2+\sqrt{6}^2}=\sqrt{12}=2\sqrt{3}$
    Argument: $$\left\{ \begin{array}{l } \cos(\theta)=\dfrac{a}{r}= \dfrac{\sqrt{6}}{2\sqrt{3}}= \dfrac{\sqrt 2}{2} \\ \sin(\theta)=\dfrac{b}{r}= \dfrac{\sqrt{6}}{2\sqrt{3}}= \dfrac{\sqrt2}{ 2} \end{array} \right.$$ Ainsi $\theta=\dfrac{\pi}{4}$ convient; on a donc: $$z_1=[2\sqrt{3};\dfrac{\pi}{4}] \text{ ou } z_1=2\sqrt{3}\left [\cos\left (\dfrac{\pi}{4}\right )+i\sin\left (\dfrac{\pi}{4}\right )\right ]$$
    La forme exponentielle de $z_{1}$ est $z_1= 2\sqrt{3}e^{i\frac{\pi}{4}}$
  3. On considère les nombres complexes $z_{1} = \sqrt{6} + \text{i}\sqrt{6}$ et $z_{2} = - \sqrt{6} + \text{i}\sqrt{6}$. Le nombre complexe $z_{2}$ est égal à :
    1. $\overline{z_{1}}$
    2. $- z_{1}$
    3. $- \overline{z_{1}}$
    4. $\text{i} + z_{1}$
  4. $$z_2=-\left(\sqrt{6} - \text{i}\sqrt{6}\right)=- \overline{z_{1}}$$
    $z_2=-\overline{z_1}$
  5. La fonction $f$ est définie sur l'intervalle $]0 ; +\infty[$ par $f(x) = \dfrac{1}{x}$. Sa courbe représentative est donnée ci-dessous :

    Le domaine du plan défini comme l'ensemble des points $M$ de coordonnées $(x ; y)$ qui vérifient $1 \leqslant x \leqslant 2$ et $\dfrac{1}{x} \leqslant y \leqslant 1$ a pour aire (exprimée en unité d'aire) :
    1. $\ln 2$
    2. $\dfrac{1}{2}$
    3. $1 - \ln 2$
    4. $1 - \text{e}^2$
  6. Le domaine du plan défini comme l'ensemble des points $M$ de coordonnées $(x;y)$ qui vérifient $1⩽x⩽2$ et $\dfrac{1}{x}\leq y\leq1$ est le domaine du plan compris entre la droite d'équation $y=1$ , la courbe représentative de la fonction $f$ et les droites d'équation $x=1$ et $x=2$ . L'aire, exprimée en unité d'aire, de ce domaine est donc égale à la différence entre l'aire du carré de côté 1 et l’intégrale de la fonction $f$ sur l'intervalle $[1;2]$ : $$\begin{array}{ll} \mathcal{A}&= 1 -\int_1^2 \dfrac{1}{x} \; dx\\ &= 1- \left[ \ln x \right]_1^2\\ &= 1-\left( \ln 2-\ln 1 \right)\\ &= 1 -\ln 2\end{array}$$
    $\mathcal{A} =1-\ln 2$
  7. La tangente au point d'abscisse $\dfrac{1}{2}$ à la courbe représentative de la fonction $f$, définie sur l'intervalle $]0 ; + \infty[$ par $f(x) = \dfrac{1}{x}$, a pour équation :
    1. $y = - 4x + 4$
    2. $y = \phantom{-}4x + 4$
    3. $y = - 4x - 4$
    4. $y = \phantom{-}4x - 4$
  8. La tangente au point d'abscisse $\dfrac{1}{2}$ à la courbe représentative de la fonction $f$ a pour équation : $$y=f’\left(\dfrac{1}{2}\right)\times \left(x−\dfrac{1}{2}\right)+f\left(\dfrac{1}{2}\right)$$ Or $f’$ est la fonction définie sur l'intervalle $]0;+\infty[$ par $f'(x)=−\dfrac{1}{x^2}$ d'où $f’\left(\dfrac{1}{2}\right)=-4$. D'autre part, $f\left(\dfrac{1}{2}\right)=2$. Donc la tangente au point d'abscisse $\dfrac{1}{2}$ à la courbe représentative de la fonction $f$ a pour équation : $$y=-4\times \left(x−\dfrac{1}{2}\right)+2\iff y=-4x+4$$
    Donc la tangente au point d'abscisse $\dfrac{1}{2}$ à la courbe représentative de la fonction $f$ a pour équation :$y=-4x+4$

Exercice 2 5 points


Fonction ln


Une équipe aérospatiale se propose d'envoyer un satellite de $10$ tonnes en orbite autour de la Terre par l'intermédiaire d'une fusée à un seul étage. Cette fusée a une masse à vide, c'est-à-dire sans carburant ni satellite, de $40$ tonnes. L'éjection des gaz permet à la fusée de décoller et de s'élever dans les airs jusqu'à la consommation totale du propergol, carburant contenu dans ses réservoirs. La vitesse d'éjection des gaz est $V_{\text{e}} = 3200 $ m.s$^{-1}$. La vitesse finale de la fusée vitesse atteinte lorsque les réservoirs sont vides, varie en fonction de la masse de propergol contenue au départ dans les réservoirs. Elle doit être de $8000$ m.s$^{-1}$ pour permettre la mise en orbite souhaitée. Le but de l'exercice est de déterminer la masse de propergol à mettre dans les réservoirs pour permettre cette mise en orbite du satellite. On note $x$ la masse, en tonnes, de propergol contenu au décollage dans les réservoirs de la fusée. La masse $x$ est comprise entre 100 et 900 tonnes. La masse totale de la fusée est alors $(x + 50)$ tonnes. Il est établi que la vitesse finale de la fusée, $f(x)$, exprimée en m· s$^{-1}$, est donnée par \[f(x) = V_{\text{e}} \times [\ln(x + 50) - \ln 50]\] où $x$ est un réel de l'intervalle [100 ; 900].


  1. Montrer que, pour tout réel $x$ de l'intervalle [100 ; 900], $f(x) = 3200 \times \ln (0,02x + 1)$. On pourra choisir l'une ou l'autre des expressions de $f(x)$ pour répondre à chacune des questions suivantes.
    1. Si les réservoirs contiennent au décollage 100 tonnes de propergol, quelle sera la vitesse finale de la fusée ?
    2. Avec 400 tonnes de propergol au décollage la mise en orbite sera-t-elle possible ?
    1. Calculer la fonction dérivée $f'$ de la fonction $f$.
    2. En déduire le sens de variation de la fonction $f$.
  2. Déterminer la masse de propergol à mettre dans les réservoirs pour permettre la mise en orbite souhaitée.

 


Correction de l'exercice 2 (5 points)


Fonction ln


Une équipe aérospatiale se propose d'envoyer un satellite de $10$ tonnes en orbite autour de la Terre par l'intermédiaire d'une fusée à un seul étage. Cette fusée a une masse à vide, c'est-à-dire sans carburant ni satellite, de $40$ tonnes. L'éjection des gaz permet à la fusée de décoller et de s'élever dans les airs jusqu'à la consommation totale du propergol, carburant contenu dans ses réservoirs. La vitesse d'éjection des gaz est $V_{\text{e}} = 3200 $ m.s$^{-1}$. La vitesse finale de la fusée vitesse atteinte lorsque les réservoirs sont vides, varie en fonction de la masse de propergol contenue au départ dans les réservoirs. Elle doit être de $8000$ m.s$^{-1}$ pour permettre la mise en orbite souhaitée. Le but de l'exercice est de déterminer la masse de propergol à mettre dans les réservoirs pour permettre cette mise en orbite du satellite. On note $x$ la masse, en tonnes, de propergol contenu au décollage dans les réservoirs de la fusée. La masse $x$ est comprise entre 100 et 900 tonnes. La masse totale de la fusée est alors $(x + 50)$ tonnes. Il est établi que la vitesse finale de la fusée, $f(x)$, exprimée en m· s$^{-1}$, est donnée par \[f(x) = V_{\text{e}} \times [\ln(x + 50) - \ln 50]\] où $x$ est un réel de l'intervalle [100 ; 900].


  1. Montrer que, pour tout réel $x$ de l'intervalle [100 ; 900], $f(x) = 3200 \times \ln (0,02x + 1)$. On pourra choisir l'une ou l'autre des expressions de $f(x)$ pour répondre à chacune des questions suivantes.
  2. $f(x)=V_e\times \left[\ln(x+50)−\ln50\right]$ avec $Ve=3200m.s^{-1}$ d'où : $$\begin{array}{ll} f(x)&= 3200\times \left[\ln(x+50)−\ln50\right]\\ &=3200\times \ln\left(\dfrac{x+50}{50}\right)\\ & =3200\times \ln(0,02x+1) \end{array}$$
    $f$ est la fonction définie pour tout réel $x$ de l'intervalle $[100;900]$ par $f(x)=3200\times \ln(0,02x+1).$
    1. Si les réservoirs contiennent au décollage 100 tonnes de propergol, quelle sera la vitesse finale de la fusée ?
    2. $f(100)=3200\times\ln 3\approx 3515,6$
      Avec 100 tonnes de propergol au décollage, la vitesse finale de la fusée sera d'environ $3516m.s^{-1}.$
    3. Avec 400 tonnes de propergol au décollage la mise en orbite sera-t-elle possible ?
    4. $f(400)=3200\times \ln9\approx 7031,1$
      Avec 400 tonnes de propergol au décollage, la vitesse finale de la fusée ne permettra pas la mise en orbite du satellite.
    1. Calculer la fonction dérivée $f'$ de la fonction $f$.
    2. $$\begin{array}{ll} f’(x)&= 3200\times \dfrac{0,02}{0,02x+1}\\ &\dfrac{64}{0,02x+1}\\ \end{array}$$
      $f’$ est la fonction définie pour tout réel $x$ de l'intervalle $[100;900]$ par $f’(x)=\dfrac{64}{0,02x+1}.$
    3. En déduire le sens de variation de la fonction $f$.
    4. Pour tout réel $x$ de l'intervalle $[100;900]$, $0,02x+1>0 $ donc $f’(x)>0$.
      La dérivée $f’$ est positive sur l'intervalle $[100;900]$ donc la fonction $f $ est croissante.
  3. Déterminer la masse de propergol à mettre dans les réservoirs pour permettre la mise en orbite souhaitée.
  4. $$\begin{array}{ll} 3200\times \ln(0,02x+1)=8000&\iff \ln(0,02x+1)=2,5\\ &\iff 0,02x+1=e^{2,5}\\ &\iff 0,02x=e^{2,5}-1\\ &\iff x=\dfrac{e^{2,5}-1}{0,02}\\ &\iff x=50\left(e^{2,5}-1\right)\\ \end{array}$$ Comme $50\left(e^{2,5}-1\right)\approx 559,1$ et que la fonction $f$ est croissante :
    $560$ tonnes de de propergol sont nécessaires pour permettre la mise en orbite du satellite.

 

 


Exercice 3 5 points


Suites


Chloé, âgée de 15 ans au 1er janvier 2014, réside dans une agglomération française. Pour anticiper le financement de son permis de conduire, elle décide de placer sur un produit d'épargne ses $600$ euros d'économies à partir du 1er janvier 2014.


Information 1 : conditions de souscription du livret jeune
  • Montant maximum de placement: 1600 euros
  • Taux d'intérêt annuel de 2,75$\,\%$
  • Avoir entre 12 et 25 ans
  • Montant minimum à l'ouverture : 10 euros
  • Résider en France
Information 2 : coût moyen du permis de conduire
  • La loi impose un minimum de $20$ heures de conduite avant de se présenter au permis.
  • Une enquête de la CLCV (Consommation, Logement et Cadre de Vie) publiée en août 2013 et menée auprès de $447$ auto-écoles souligne que ce forfait de $20$ heures est facturé du simple au double selon les régions.
  • Par ailleurs, même si le minimum imposé par la loi est de vingt heures de conduite, il en faut plutôt trente en moyenne.
  • Ainsi, en comptant les frais de dossier, il est préférable de prévoir un budget de 1500 euros.
Partie A

  1. Expliquer pourquoi Chloé remplit les conditions permettant de souscrire au livret jeune.
  2. Aura-t-elle une somme suffisante disponible au 1er janvier 2017 pour passer son permis si elle choisit de souscrire au livret jeune ?

Partie B

Chloé aura besoin de 1500 euros pour financer son permis. Ses parents lui conseillent de verser chaque mois sur le livret la somme supplémentaire de 25 euros, à partir du 1er février 2014. Ils lui expliquent que le taux annuel du livret jeune correspond à un taux mensuel de 0,226$\,\%$.


  1. Ses parents lui présentent un extrait d'une page de tableur qui simule l'évolution d'épargne : $$ \begin{array}{|c|c|} \hline &A& B\\ \hline 1 &01/01/2014 &600,00 \text{€}\\ \hline 2 &01/02/2014 &626,36 \text{€}\\ \hline 3 &01/03/2014 &652,77 \text{€}\\ \hline 4 &01/04/2014 &679,25 \text{€}\\ \hline 5 &01/05/2014 &705,78 \text{€}\\ \hline 6 &01/06/2014 &732,38 \text{€}\\ \hline 7 &... &...\\ \hline \end{array}$$
    1. Justifier que, dans la feuille de calcul ci-dessus, la formule à saisir dans la cellule $B2$ est

        $=  1,00226 \times \text{B}1 + 25$.
    2. Déterminer la somme qui serait disponible sur le livret au 1er juillet 2014.
  2. Chloé veut déterminer au bout de combien de mois elle aurait l'argent nécessaire pour financer son permis en suivant le conseil de ses parents. Elle décide de noter $u_{n}$ la somme, en euros, disponible le $n$-ième mois après l'ouverture du livret. Ainsi, $u_{0}$ vaut 600 euros.
    1. Exprimer $u_{n+1}$ en fonction de $u_{n}$.
    2. Chloé décide d'écrire l'algorithme suivant : $$\begin{array}{|ll|}\hline\text{Variables}\\ \hspace{0,3cm}n :\text{ un nombre entier naturel}\\ \hspace{0,3cm}u : \text{ un nombre réel }\\\text{Initialisation}\\ \hspace{0,3cm}\text{ Affecter à } n \text{ la valeur } 0\\ \hspace{0,3cm}\text{ Affecter à } u \text{ la valeur } 600\\\text{Traitement}\\ \hspace{0,3cm} \text{ Tant que }\ldots\ldots\\ \hspace{0,6cm}\text{ Affecter à } n \text{ la valeur } n + 1\\ \hspace{0,6cm}\text{ Affecter à } u \text{ la valeur }\ldots\ldots\\ \hspace{0,3cm}\text{ Fin Tant que }\\\text{Sortie}\\ \hspace{0,3cm}\text{ Afficher }\ldots\\ \hline \end{array} $$ Trois lignes de l'algorithme comportent des pointillés. Recopier ces lignes et les compléter pour que Chloé puisse déterminer le nombre de mois cherché.
    3. Au bout de combien de mois Chloé aura-t-elle l'argent nécessaire pour financer son permis si elle suit les conseils de ses parents ?

 


Correction de l'exercice 3 (5 points)


Suites


Chloé, âgée de 15 ans au 1er janvier 2014, réside dans une agglomération française. Pour anticiper le financement de son permis de conduire, elle décide de placer sur un produit d'épargne ses $600$ euros d'économies à partir du 1er janvier 2014.


Information 1 : conditions de souscription du livret jeune
  • Montant maximum de placement: 1600 euros
  • Taux d'intérêt annuel de 2,75$\,\%$
  • Avoir entre 12 et 25 ans
  • Montant minimum à l'ouverture : 10 euros
  • Résider en France
Information 2 : coût moyen du permis de conduire
  • La loi impose un minimum de $20$ heures de conduite avant de se présenter au permis.
  • Une enquête de la CLCV (Consommation, Logement et Cadre de Vie) publiée en août 2013 et menée auprès de $447$ auto-écoles souligne que ce forfait de $20$ heures est facturé du simple au double selon les régions.
  • Par ailleurs, même si le minimum imposé par la loi est de vingt heures de conduite, il en faut plutôt trente en moyenne.
  • Ainsi, en comptant les frais de dossier, il est préférable de prévoir un budget de 1500 euros.
Partie A

  1. Expliquer pourquoi Chloé remplit les conditions permettant de souscrire au livret jeune.
  2. Chloé, âgée de 15 ans au 1er janvier 2014, réside dans une agglomération française et verse 600 euros. Chloé remplit les conditions permettant de souscrire au livret jeune.
  3. Aura-t-elle une somme suffisante disponible au 1er janvier 2017 pour passer son permis si elle choisit de souscrire au livret jeune ?
  4. Soit $C_n$ le capital disponible le 1er janvier de l'année $(2014+n)$.
    Le taux d'intérêt annuel étant de $2,75\ %$, on a :
    $C_{n+1}=C_n\times (1+\dfrac{2,75}{100})=1,0275\times C_n$
    Ainsi, $(C_n)$ est une suite géométrique de raison $1,0275$ et de premier terme $C_0=600$
    donc pour tout entier $n, C_n=600\times 1,0275^n$
    Le capital disponible au 1er janvier 2017 est : $$C_3=600\times1,0275^3\approx 650,87$$
    Au 1er janvier 2017, Chloé n'aura pas une somme suffisante pour passer son permis.

Partie B

Chloé aura besoin de 1500 euros pour financer son permis. Ses parents lui conseillent de verser chaque mois sur le livret la somme supplémentaire de 25 euros, à partir du 1er février 2014. Ils lui expliquent que le taux annuel du livret jeune correspond à un taux mensuel de 0,226$\,\%$.


  1. Ses parents lui présentent un extrait d'une page de tableur qui simule l'évolution d'épargne : $$ \begin{array}{|c|c|} \hline &A& B\\ \hline 1 &01/01/2014 &600,00 \text{€}\\ \hline 2 &01/02/2014 &626,36 \text{€}\\ \hline 3 &01/03/2014 &652,77 \text{€}\\ \hline 4 &01/04/2014 &679,25 \text{€}\\ \hline 5 &01/05/2014 &705,78 \text{€}\\ \hline 6 &01/06/2014 &732,38 \text{€}\\ \hline 7 &... &...\\ \hline \end{array}$$
    1. Justifier que, dans la feuille de calcul ci-dessus, la formule à saisir dans la cellule $B2$ est 
      $=  1,00226 \times \text{B}1 + 25$
    2. Le coefficient multiplicateur associé à un pourcentage d'évolution de $0,226 \%$ est $1+\dfrac{0,226}{100}=1,00226$.
      La somme disponible chaque mois est obtenue en ajoutant 25 au produit de la somme disponible du mois précédent par 1,00226 donc la formule à saisir dans la cellule $B2$ est : $$=1,00226\times B1+25.$$
    3. Déterminer la somme qui serait disponible sur le livret au 1er juillet 2014.
    4. $732,38\times 1,00226 +25 \approx 759,04.$
      Au 1er juillet 2014, elle disposerait de 759,04 euros.
  2. Chloé veut déterminer au bout de combien de mois elle aurait l'argent nécessaire pour financer son permis en suivant le conseil de ses parents. Elle décide de noter $u_{n}$ la somme, en euros, disponible le $n$-ième mois après l'ouverture du livret. Ainsi, $u_{0}$ vaut 600 euros.
    1. Exprimer $u_{n+1}$ en fonction de $u_{n}$.
    2. Pour tout entier naturel $n, u_{n+1}=1,00226\times u_n+25.$
    3. Chloé décide d'écrire l'algorithme suivant : Trois lignes de l'algorithme comportent des pointillés. Recopier ces lignes et les compléter pour que Chloé puisse déterminer le nombre de mois cherché.
    4. $$\begin{array}{|ll|}\hline\text{Variables}\\ \hspace{0,3cm}n :\text{ un nombre entier naturel}\\ \hspace{0,3cm}u : \text{ un nombre réel }\\\text{Initialisation}\\ \hspace{0,3cm}\text{ Affecter à } n \text{ la valeur } 0\\ \hspace{0,3cm}\text{ Affecter à } u \text{ la valeur } 600\\\text{Traitement}\\ \hspace{0,3cm} \text{ Tant que } u < 1500 \\ \hspace{0,6cm}\text{ Affecter à } n \text{ la valeur } n + 1 \\ \hspace{0,6cm}\text{ Affecter à } u \text{ la valeur }1,00226\times u+25\\ \hspace{0,3cm}\text{ Fin Tant que }\\\text{Sortie}\\ \hspace{0,3cm}\text{ Afficher }n\\ \hline \end{array} $$
    5. Au bout de combien de mois Chloé aura-t-elle l'argent nécessaire pour financer son permis si elle suit les conseils de ses parents ?
    6. On programme l'algorithme sur la calculatrice, on obtient en sortie 33.
      Chloé aura l'argent nécessaire pour financer son permis au bout de 33 mois.

 


Exercice 4 5 points


Probabilités


Une entreprise de transport dispose d'un nombre important de camions. On admet que la distance quotidienne parcourue par chaque camion, exprimée en kilomètres, peut être modélisée par une variable aléatoire $X$ qui suit la loi normale d'espérance $500$ et d'écart type $40$.


  1. Donner la distance moyenne parcourue en un jour par un camion.
  2. Déterminer la probabilité qu'un camion parcoure au moins $500$ km en un jour.
  3. Déterminer la probabilité qu'un camion parcoure entre $380$ km et $460$ km en un jour.
  4. Déterminer la probabilité qu'un camion parcoure plus de $460$ km en un jour.
  5. Le directeur de l'entreprise affirme qu'environ 95$\,\%$ de ses camions parcourent entre $460$ et $540$ km par jour. A-t-il raison ?

Exercice 4 5 points


Probabilités


Une entreprise de transport dispose d'un nombre important de camions. On admet que la distance quotidienne parcourue par chaque camion, exprimée en kilomètres, peut être modélisée par une variable aléatoire $X$ qui suit la loi normale d'espérance $500$ et d'écart type $40$.


  1. Donner la distance moyenne parcourue en un jour par un camion.
  2. La distance quotidienne parcourue par chaque camion, est modélisée par une variable aléatoire X qui suit la loi normale d'espérance 500 donc :
    La distance moyenne parcourue en un jour par un camion est de 500 km.
  3. Déterminer la probabilité qu'un camion parcoure au moins $500$ km en un jour.
  4. La variable aléatoire $X$ suit la loi normale d'espérance 500 donc $P(X\geq 500)=0,5$
    La probabilité qu'un camion parcoure au moins 500 km en un jour est égale à 0,5.
  5. Déterminer la probabilité qu'un camion parcoure entre $380$ km et $460$ km en un jour.
  6. 2ND DISTR 2NORMALFRép( \1 , \2,\3,\4)EXE
    Avec une calculatrice de type TI

    $$NormalFR\text{é}p(\1,\2,\3,\4) \approx \5$$

    $$P(\1 \leq \6 \leq \2)\approx \5 \text{ à } 10^{-\7} \text{ près.}$$

     

    La probabilité qu'un camion parcoure entre 380 km et 460 km en un jour est, arrondie au millième près, 0,157.
  7. Déterminer la probabilité qu'un camion parcoure plus de $460$ km en un jour.
  8.  

    2ND DISTR 2NORMALFRép( $\1$ , $10^{99}$,\2,$\3$)EXE
    Avec une calculatrice de type TI

    $$NormalFR\text{é}p(\1,10^{99},\2,\3) \approx \4$$

    $$P( \5 \geq \1)\approx \4 \text{ à } 10^{-\6} \text{ près.}$$
    La probabilité qu'un camion parcoure plus de 460 km en un jour est, arrondie au millième près, 0,841.
  9. Le directeur de l'entreprise affirme qu'environ 95$\,\%$ de ses camions parcourent entre $460$ et $540$ km par jour. A-t-il raison ?
  10. Si $X$ suit la loi normale d'espérance $\mu$ et d'écart-type $\sigma$ alors $P(\mu-\sigma\leq X\leq \mu+\sigma)≈0,683$ d'où :
    $P(460\leq X\leq 540)\approx 0,683$ par conséquent, le directeur a tort.
  • Vues: 10834

Baccalauréat STI2D Antilles-Guyane 18 juin 2014

 Sciences et technologies du design et des arts appliqués

Exercice 1 (7 points)


Fonctions

Un designer-graphiste a imaginé le logo ci-dessous. Il est constitué de deux demi-cercles concentriques et d'une courbe. L'objectif de cet exercice est de reproduire ce logo.

Antilles-Guyane 18 juin 2014-Ex1

Partie A : Les demi-cercles

  1. Dans le repère orthonormal $\left(\text{O},~\vec{i},\vec{j}\right)$ de l'annexe 1, on a placé les points A(5 ; 2) et B$(-1 ; 2)$ puis on a tracé le demi-cercle supérieur joignant les points A et B.
    1. Donner une équation cartésienne du cercle de diamètre [AB].
    2. Déterminer par le calcul les coordonnées exactes du point d'intersection de l'axe des ordonnées avec ce demi-cercle.
    1. Sur la figure de l'annexe 1, placer le point C(4 ; 4) puis tracer le demi-cercle joignant les points C et O, correspondant au deuxième demi-cercle du logo.
    2. Tracer la droite $(T)$, tangente à ce demi-cercle au point O. Déterminer graphiquement le coefficient directeur de $(T)$.

Partie B : La courbe
On souhaite construire la base du logo avec un raccordement lisse en O. Soit $f$ la fonction définie sur [0 ; 5] par \[f(x) = - 0,072 x^3 + 0,64 x^2 - x.\] On note $\mathcal{C}_{f}$ la courbe représentant la fonction $f$ dans le repère de l'annexe 1.

  1. Justifier que la courbe $\mathcal{C}_{f}$ passe par les points 0 et A.
  2. On note l' la fonction dérivée de la fonction $f$.
    1. Calculer $f'(x)$.
    2. Déterminer le coefficient directeur de la tangente à la courbe $\mathcal{C}_{f}$ au point d'abscisse $0$.
    3. Calculer $f'(5)$ et donner une interprétation graphique du résultat obtenu.
    4. Vérifier que $f'(x) = 0,008 (5 - x)(27x - 25)$.
  3. En déduire les variations de la fonction $f$ sur l'intervalle [0 ; 5].
  4. Sur l'annexe 1, compléter le tableau de valeurs de la fonction $f$ (on arrondira à $10^{-3}$ près).
  5. Sur l'annexe 1, compléter le logo en traà§ant $\mathcal{C}_{f}$

Correction de l'exercice 1 (7 points)


Fonctions

Exercice 2 5 points

 



QCM

 

Cet exercice est un questionnaire à choix multiples. Pour chaque question, une seule réponse est exacte. Le candidat indiquera sur sa copie le numéro de la question suivi de la réponse choisie sans aucune justification.

Une réponse exacte rapportera $1$ point. Une fausse réponse ne rapportera aucun point.

  1. Le niveau d'intensité sonore, exprimé en décibels (dB), est donné par la formule : \[L = 10 \log \left(\dfrac{I}{I_{0}} \right)\] où $I$ représente l'intensité sonore exprimée en W/m$^2$ et $I_{0}$ une intensité sonore de référence. Si $L = 70$ dB, alors :
    1. $I = 10^{-7}I_{0}$
    2. $I = = 7I_{0}$
    3. $I = = 10^{7}I_{0}$
    4. $I = \log (60)I_{0}$
  2. On considère le cube ABCDEFGH représenté dessous.
    Antilles-Guyane 18 juin 2014-Ex2 cube
    Le triangle EBD est :
    1. quelconque
    2. isocèle non équilatéra
    3. rectangle
    4. équilatéral
  3. Le cube précédent est invariant par rotation d'axe (AG) et d'angle :
    1. 60 °
    2. 90 °
    3. 120 °
    4. 180 °
  4. On a représenté en perspective centrale le carrelage ci-dessous :
    Antilles-Guyane 18 juin 2014-Ex2 carrelage
    (h) désignant la ligne de fuite du plan contenant ce carrelage, dans quel cas les règles de perspective centrale ont-elles été respectées ?
    Antilles-Guyane 18 juin 2014-Ex2-q4
  5. On considère l'ellipse dont l'équation réduite est : \[\dfrac{(x + 2)^2}{9} + \dfrac{(y - 1)^2}{4} = 1.\] Cette ellipse est représentée dans un repère orthonormal par :
    Antilles-Guyane 18 juin 2014-Ex2-q5
  6. Dans un repère orthonormal de l'espace, on considère les vecteurs: \[\vec{u}(5 ; 1 ; 0) \quad \text{et} \quad \vec{v}(1 ; -3 ; 1).\] Le produit scalaire $\vec{u} \cdot \vec{v}$ vaut :
    1. $- 1$
    2. $0$
    3. $1$
    4. $2$

Exercice 3 7 points


Pavages

Les trois parties peuvent être traitées de manière indépendante

Partie A : observation du pavage

A l'aide d'un logiciel de géométrie dynamique, on a construit le pavage donné en annexe 2. Ce pavage est constitué d'hexagones identiques.

  1. Quelle transformation permet de passer de l'hexagone 1 à l'hexagone 2. Préciser les caractéristiques de cette transformation.
  2. Hachurer sur le pavage de l'annexe 2 tous les hexagones qui sont l'image de l'hexagone 1 par une translation.

Partie B : Obtention du motif

  • On considère un triangle équilatéral ABC de cà´té 5 cm et un point M situé à l'intérieur du triangle ABC tel que AM = 4 cm et BM = 2 cm.
  • Le point M$_{1}$ est l'image du point M par la symétrie axiale d'axe (AB).
  • Le point M$_{2}$ est l'image du point M par la symétrie axiale d'axe (BC).
  • Le point M$_{3}$ est l'image du point M par la symétrie axiale d'axe (AC).
    • Antilles-Guyane 18 juin 2014-EX3-f1

      1. Justifier que l'hexagone AM$_{1}$BM$_{2}$CM$_{}$ a pour aire le double de l'aire du triangle ABC.
        1. En appliquant la formule d'Al-Kashi dans le triangle MAB, déterminer la valeur exacte du cosinus de l'angle $\widehat{\text{MAB}}$ puis la mesure approchée au dixième de degré de cet angle. En déduire la mesure approchée au dixième de degré de l'angle $\widehat{\text{MAC}}$.
        2. Dans cette question, on prendra 37,7 °{} comme mesure de l'angle $\widehat{\text{MAC}}$. Déterminer par le calcul une valeur approchée au centième de centimètre près de la longueur CM.

      Partie C : Construction d'un pavage différent

      L'objectif de cette partie est de construire un pavage différent.
      On place cette fois le point M à l'extérieur du triangle équilatéral.
      Sur l'annexe 3, un triangle équilatéral ABC est tracé. M est un point extérieur au triangle.

      1. Construire le symétrique M$_{1}$ de M par rapport à l'axe (AB), le symétrique M$_{2}$ de M par rapport à l'axe (BC), le symétrique M$_{3}$ de M par rapport à l'axe (AC). Tracer en couleur l'hexagone AM$_{1}$BM$_{2}$CM$_{3}$.
      2. En utilisant des couleurs différentes, construire soigneusement l'image de cet hexagone par la rotation de centre A et d'angle 120 °, puis par la rotation de centre A et d'angle 240 °, le sens de rotation choisi étant le sens anti-horaire (le sens inverse des aiguilles d'une montre). Laisser les traits de construction apparents.

      Annexe 1 - Exercice 1 (à rendre avec la copie)

      Partie A

      Antilles-Guyane 18 juin 2014-Annexe1

      Partie B

      $$\begin{array}{ |c|c|c|c|c|c|c|} \hline x&0&1&2&3&4&5\\ \hline f(x)&&&- 0,016 &&1,632&\\ \hline \end{array}$$

      Annexe 2 - Exercice 3 (à rendre avec la copie)

      Antilles-Guyane 18 juin 2014-Annexe2-Ex3

      Annexe 3 - Exercice 3 (à rendre avec la copie)

      STI2DA Antilles Juin 2014 triangle-ABC

      • Vues: 7800

      Baccalauréat STI 2D/STL spécialité SPCL Antilles-Guyane 19 juin 2014

      Exercice 1 5 points


      QCM nombres complexes, suites et équations différentielles

      Cet exercice est un questionnaire à choix multiples. Pour chacune des questions suivantes, une seule des quatre réponses proposées est exacte. Aucune justification n'est demandée. Une bonne réponse rapporte un point. Une mauvaise réponse, plusieurs réponses ou l'absence de réponse à une question ne rapportent ni n'enlèvent aucun point. Indiquer sur la copie le numéro de la question et la réponse correspondante choisie.

      1. Soit $x$ un réel quelconque, $\text{e}^{- 4x}$ est égal à :
        1. $\text{e}^{x} \times \text{e}^{- 4}$
        2. $- \text{e}^{4x}$
        3. $x \times \text{e}^{- 4}$
        4. $\dfrac{1}{\text{e}^{4x}}$
      2. L'intégrale $\displaystyle\int_{\ln 2}^{\ln 3}\text{e}^{2x}\:\text{d}x$ est égale à :
        1. 5
        2. 10
        3. 2,5
        4. 1
      3. $\left(u_{n}\right)$ est la suite géométrique de premier terme $u_{0} = 5$ et de raison $0,98$. $\left(v_{n}\right)$ est la suite géométrique de premier terme $v_{0} = 2,8$ et de raison $1,02$. Le plus petit entier $n$ vérifiant $u_{n} \leqslant v_{n}$ est :
        1. 14
        2. 15
        3. 16
        4. 17
      4. $\left(u_{n}\right)$ est la suite géométrique de premier terme $u_{0} = 1$ et de raison $\dfrac{5}{3}$. On donne l'algorithme suivant : $$\begin{array}{|l |l|}\hline \text{Variables} & n, u\\ \text{Initialisation} & u \text{ prend la valeur 1}\\ &n \text{ prend la valeur 0}\\ \text{Traitement} & \text{ Tant que } u < 1000 \\ &\quad n \text{ prend la valeur } n + 1\\ &\quad u \text{ prend la valeur } u \times \dfrac{5}{3}\\ &\text{ Fin tant que}\\ \text{Sortie} & \text{ Afficher } n\\ \hline \end{array}$$ Cet algorithme affiche en sortie :
        1. la valeur de $u_{ 1001 }$
        2. la plus grande valeur de $n$ vérifiant $u_{n} < 1000$
        3. la plus petite valeur de $n$ vérifiant $u_{n} \geqslant 1000$
        4. la plus petite valeur de $u_{n}$ vérifiant $u_{n} \geqslant 100 $
      5. Soit $f$ la fonction définie sur $\mathbb{R}$ par $f(x) = 2 \cos \left(\dfrac{4}{3}x - \dfrac{\pi}{6}\right)$. La fonction $f$ est une solution de l'équation différentielle :
        1. $y"" + y = 0$
        2. $16y'' - 9y = 0$
        3. $9y'' + 16y = 0$
        4. $9y'' - 16y = 0$

      Correction de l'exercice 1 (5 points)


      QCM nombres complexes, suites et équations différentielles

      Cet exercice est un questionnaire à choix multiples. Pour chacune des questions suivantes, une seule des quatre réponses proposées est exacte. Aucune justification n'est demandée. Une bonne réponse rapporte un point. Une mauvaise réponse, plusieurs réponses ou l'absence de réponse à une question ne rapportent ni n'enlèvent aucun point. Indiquer sur la copie le numéro de la question et la réponse correspondante choisie.

        Pour tout réel $x$ on a $\text{e}^{- x}=\dfrac{1}{\text{e}^{x}}$
      1. Soit $x$ un réel quelconque, $\text{e}^{- 4x}$ est égal à :
        1. FAUX
        2. FAUX
        3. FAUX
        4. VRAI : $\dfrac{1}{\text{e}^{4x}}$
      2. L'intégrale $\displaystyle\int_{\ln 2}^{\ln 3}\text{e}^{2x}\:\text{d}x$ est égale à : $$\begin{array}{ll} I&= \displaystyle\int_{\ln 2}^{\ln 3}\text{e}^{2x}\:\text{d}x\\ & = \left [ \dfrac{1}{2} \text{e}^{2x}\right ]_{\ln 2 }^{\ln 3} \\ & = \dfrac{1}{2} \text{e}^{2\ln 3} - \dfrac{1}{2} \text{e}^{2\ln 2} \\ & = \dfrac{1}{2} \text{e}^{\ln 9} - \dfrac{1}{2} \text{e}^{\ln 4} \\ & = \dfrac{9}{2} - \dfrac{4}{2} \\ & = \dfrac{5}{2} \\ \end{array}$$
        1. FAUX
        2. FAUX
        3. VRAI $I=$2,5
        4. FAUX
      3. $\left(u_{n}\right)$ est la suite géométrique de premier terme $u_{0} = 5$ et de raison $0,98$. $\left(v_{n}\right)$ est la suite géométrique de premier terme $v_{0} = 2,8$ et de raison $1,02$. Le plus petit entier $n$ vérifiant $u_{n} \leqslant v_{n}$ est : $$\begin{array}{ll} u_n\leq v_n &\iff 5 \times 0,98 ^n \leq 2,8 \times 1,02^n &\\ & \iff \dfrac{0,98^n}{1,02^n} \leq \dfrac{2,8}{1,02} & u_n=q ^n \times u_0\\ & \iff \left (\dfrac{0,98}{1,02}\right )^n \leq 0,56 & \\ & \iff \ln \left ( \left (\dfrac{0,98}{1,02}\right )^n \right ) \leq \ln (0,56) & \text{ On apllique } \ln \\ & \iff n\ln \left ( \dfrac{0,98}{1,02} \right ) \leq \ln (0,56) & \text{strictement croissante sur } ]0;+\infty[ \\ & \iff n \geq \dfrac{ \ln (0,56) }{\ln \left ( \dfrac{0,98}{1,02} \right )} & \text{car } \ln \left ( \dfrac{0,98}{1,02} \right ) < 0 \\ \end{array}$$ Par ailleurs $ \dfrac{ \ln (0,56) }{\ln \left ( \dfrac{0,98}{1,02} \right )}\approx 14,5$
        Le plus petit entier $n$ vérifiant $u_{n} \leqslant v_{n}$ est 15.
        1. FAUX
        2. VRAI : 15
        3. FAUX
        4. FAUX
      4. $\left(u_{n}\right)$ est la suite géométrique de premier terme $u_{0} = 1$ et de raison $\dfrac{5}{3}$. On donne l'algorithme suivant : $$\begin{array}{|l |l|}\hline \text{Variables} & n, u\\ \text{Initialisation} & u \text{ prend la valeur 1}\\ &n \text{ prend la valeur 0}\\ \text{Traitement} & \text{ Tant que } u < 1000 \\ &\quad n \text{ prend la valeur } n + 1\\ &\quad u \text{ prend la valeur } u \times \dfrac{5}{3}\\ &\text{ Fin tant que}\\ \text{Sortie} & \text{ Afficher } n\\ \hline \end{array}$$ Cet algorithme affiche en sortie :
        1. FAUX
        2. FAUX
        3. VRAI :la plus petite valeur de $n$ vérifiant $u_{n} \geqslant 1000$
        4. FAUX
      5. Soit $f$ la fonction définie sur $\mathbb{R}$ par $f(x) = 2 \cos \left(\dfrac{4}{3}x - \dfrac{\pi}{6}\right)$. Pour tout réel $x$ on a $$f'(x)= 2 \times \dfrac{4}{3}\times \left (-\sin \left(\dfrac{4}{3}x - \dfrac{\pi}{6}\right)\right )=-\dfrac{8}{3}\sin \left(\dfrac{4}{3}x - \dfrac{\pi}{6}\right)$$ De même on calcule $$f''(x)=-\dfrac{8}{3}\times \dfrac{4}{3} \cos \left(\dfrac{4}{3}x - \dfrac{\pi}{6}\right)=-\dfrac{32}{9} \cos \left(\dfrac{4}{3}x - \dfrac{\pi}{6}\right)$$ On a alors $$9\times f''(x)+16 \times f(x)= 9\times \left (-\dfrac{32}{9} \cos \left(\dfrac{4}{3}x - \dfrac{\pi}{6}\right)\right )+16\times 2 \cos \left(\dfrac{4}{3}x - \dfrac{\pi}{6}\right)$$ soit $$9\times f''(x)+16 \times f(x)=-32 \cos \left(\dfrac{4}{3}x - \dfrac{\pi}{6}\right)+32 \cos \left(\dfrac{4}{3}x - \dfrac{\pi}{6}\right)+0$$ La fonction $f$ est une solution de l'équation différentielle $9y'' + 16y = 0$.
        1. FAUX
        2. FAUX
        3. VRAI :$9y'' + 16y = 0$
        4. FAUX

      Exercice 2 5 points

       

      Exercice 2 5 points


      Probabilités

      Dans cet exercice, on s'intéresse à deux types A et B de téléviseurs à écran plat. Les réponses aux questions 1. a., 1. b. et 1. c. seront arrondies au centième.

      1. La durée de fonctionnement, exprimée en heures, d'un téléviseur du type A, avant que survienne la première panne, est modélisée par une variable aléatoire $X$ suivant la loi exponentielle de paramètre $\lambda = 2 \times 10^{-5}$.
        1. Calculer la probabilité que la première panne survienne avant la 32000 $^e$ heure de fonctionnement.
        2. On s'intéresse à un téléviseur de type A fonctionnant chaque jour pendant 4 heures. Calculer la probabilité que la première panne d'écran ne survienne pas avant 10 ans.
        3. On prendra $1$ année = $365$ jours .
        4. Calculer la probabilité que la première panne survienne après 10000 heures et avant 40000 heures de fonctionnement.
        5. Calculer l'espérance mathématique de la variable aléatoire $X$ et en donner une interprétation.
      2. La durée de fonctionnement avant la première panne d'un téléviseur de type B est modélisée par une variable aléatoire $Y$ suivant la loi exponentielle de paramètre $\lambda'$. Une étude statistique a permis d'évaluer $P(Y \leqslant 32000 ) = 0,8$. Calculer la valeur arrondie à $10^{-5}$ de $\lambda'$.

      Exercice 2 5 points


      Probabilités

      Dans cet exercice, on s'intéresse à deux types A et B de téléviseurs à écran plat. Les réponses aux questions 1. a., 1. b. et 1. c. seront arrondies au centième.

      1. La durée de fonctionnement, exprimée en heures, d'un téléviseur du type A, avant que survienne la première panne, est modélisée par une variable aléatoire $X$ suivant la loi exponentielle de paramètre $\lambda = 2 \times 10^{-5}$.
        1. Calculer la probabilité que la première panne survienne avant la 32000 $^e$ heure de fonctionnement.

        2. $X $suivant la loi exponentielle de paramètre $\lambda = 2 \times 10^{-5}$ alors, $$\begin{array}{ll} p(X\leq 3200)&= \displaystyle\int_{0}^{3200} 2 \times 10^{-5}\text{e}^{-2 \times 10^{-5} t}\:\text{d}t\\ & = \left [ -\text{e}^{-2 \times 10^{-5} t}\right ]_{0}^{3200} \\ & =1 - \text{e}^{-2 \times 10^{-5} \times 3200} \\ & = 1- \text{e}^{-0,64} \\ & \approx 0,47 \\ \end{array}$$
          La probabilité que la première panne survienne avant la 32000$^e$ heure de fonctionnement est, arrondie au centième près, égale à 0,47.
        3. On s'intéresse à un téléviseur de type A fonctionnant chaque jour pendant 4 heures. Calculer la probabilité que la première panne d'écran ne survienne pas avant 10 ans.
        4. On prendra $1$ année = $365$ jours .
          $10\times 4 \times 365=14600 $ et $$\begin{array}{ll} p(X\geq 14 600) & =1 -p(X < 14 600 ) \\ &= 1-\displaystyle\int_{0}^{14 600} 2 \times 10^{-5}\text{e}^{-2 \times 10^{-5} t}\:\text{d}t\\ & = 1-\left [ -\text{e}^{-2 \times 10^{-5} t}\right ]_{0}^{14 600} \\ & =1-\left (1 - \text{e}^{-2 \times 10^{-5} \times 14 600} \right ) \\ & = \text{e}^{-0,292} \\ & \approx 0,75 \\ \end{array}$$
          La probabilité que la première panne d'écran ne survienne pas avant 10 ans est, arrondie au centième près, égale à 0,75.
        5. Calculer la probabilité que la première panne survienne après 10000 heures et avant 40000 heures de fonctionnement.
        6. $$\begin{array}{ll} p(10 000 X\geq 40 00)&= \displaystyle\int_{10 000}^{40 00} 2 \times 10^{-5}\text{e}^{-2 \times 10^{-5} t}\:\text{d}t\\ & = \left [ -\text{e}^{-2 \times 10^{-5} t}\right ]_{10 000}^{40 00} \\ & = -\text{e}^{-2 \times 10^{-5} \times 40 00} + \text{e}^{-2 \times 10^{-5} \times 10 00} \\ & = - \text{e}^{-0,8} + \text{e}^{-0,2} \\ & \approx 0,37 \\ \end{array}$$
          La probabilité que la première panne survienne après 10 000 heures et avant 40 000 heures de fonctionnement est, arrondie au centième près, égale à 0,37.
        7. Calculer l'espérance mathématique de la variable aléatoire $X$ et en donner une interprétation.
        8. L'espérance mathématique de la variable aléatoire $X$est $E(X) = \dfrac{1 }{\lambda}=\dfrac{1}{2 \times 10^{-5} }=\dfrac{10^5}{2}=50 000$. La durée de fonctionnement moyenne d'un téléviseur du type A est de 50000 heures.
      2. La durée de fonctionnement avant la première panne d'un téléviseur de type B est modélisée par une variable aléatoire $Y$ suivant la loi exponentielle de paramètre $\lambda'$.
        Une étude statistique a permis d'évaluer $P(Y \leqslant 32000 ) = 0,8$.
        Calculer la valeur arrondie à $10^{-5}$ de $\lambda'$.

      3. $Y$ suivant la loi exponentielle de paramètre $\lambda'$ tel que $ p(Y\leq 32 000)=0,8$ d'où : $$\begin{array}{ll} p(Y\leq 32 000)=0,8 &\iff \displaystyle\int_{0}^{32 00} \lambda'\text{e}^{-\lambda' t}\:\text{d}t =0,8\\ & \iff \left [ -\text{e}^{-\lambda' t}\right ]_{0}^{32 00} =0,8 \\ & \iff 1 -\text{e}^{-32 000 \times \lambda'} =0,8 \\ & \iff \text{e}^{-32 000 \times \lambda'} =0,2 \\ & \iff -32 000 \times \lambda' =\ln 0,2 \\ & \iff \lambda' =-\dfrac{\ln 0,2 }{3200} \\ & \iff \lambda' =\dfrac{\ln 5 }{3200} \\ & \iff \lambda' =\dfrac{\ln 5 }{3200} \\ & \iff \lambda' \approx 5 \times 10^{-5} \end{array}$$
        La valeur arrondie à  $10^{-5}$ de  $\lambda'$ est $\lambda' \approx 5 \times 10^{-5} $

      Exercice 3 5 points


      Nombres complexes

      Les parties A et B de cet exercice sont indépendantes.
      Le plan est rapporté à un repère orthonormal $\left(\text{O},~\vec{u},~\vec{v}\right)$d'unités 5 cm.
      On note i le nombre complexe de module 1 et d'argument $\dfrac{\pi}{2}$.
      Soit $z$ le nombre complexe de module 2 et d'argument $\dfrac{\pi}{3}$,
      $\overline{z}$ est le nombre complexe conjugué de $z$.

      PARTIE A

      1. Donner les écritures algébriques de $z$, de $\overline{z}$ et de $\dfrac{1}{2}\overline{z}$.
      2. On considère le nombre complexe $p = \dfrac{2 + \overline{z}}{2 - \overline{z}}$.
        1. Montrer que $p = - \text{i}\sqrt{3}$.
        2. Les points M, N et P sont les points d'affixes respectives 1, $\dfrac{1}{2}\overline{z}$ et $p$. Placer ces trois points dans le repère. Justifier l'alignement de ces trois points.

      PARTIE B

      Soit $u$ le nombre complexe défini par $u = \dfrac{1}{2}z$.

      1. Écrire $u$ sous la forme exponentielle.
        1. Donner l'écriture exponentielle puis l'écriture algébrique de $u^3$.
        2. Vérifier les relations suivantes : $u^4 = - u$ et $u^5 = - u^2$.
        3. Vérifier que $1 + u + u^2 + u^3 + u^4 + u^5 + u^6 = 1$.

      Exercice 3 5 points


      Nombres complexes

      Les parties A et B de cet exercice sont indépendantes.
      Le plan est rapporté à un repère orthonormal $\left(\text{O},~\vec{u},~\vec{v}\right)$d'unités 5 cm.
      On note i le nombre complexe de module 1 et d'argument $\dfrac{\pi}{2}$.
      Soit $z$ le nombre complexe de module 2 et d'argument $\dfrac{\pi}{3}$,
      $\overline{z}$ est le nombre complexe conjugué de $z$.

      PARTIE A

      1. Donner les écritures algébriques de $z$, de $\overline{z}$ et de $\dfrac{1}{2}\overline{z}$.

      2. $z=2e^{i\frac{\pi}{3}}=2\left(\cos\left(\frac{\pi}{3}\right)+i\sin\left(\frac{\pi}{3}\right)\right)=2\left(\dfrac{1}{2}+i\dfrac{\sqrt 3}{2}\right)= 1 + i \sqrt 3$
        $$z= 1 + i \sqrt 3$$ $$\overline{z}= 1 - i \sqrt 3$$ $$\dfrac{1}{2}\overline{z}= \dfrac{1}{2}-i\dfrac{\sqrt 3}{2}$$
      3. On considère le nombre complexe $p = \dfrac{2 + \overline{z}}{2 - \overline{z}}$.
        1. Montrer que $p = - \text{i}\sqrt{3}$.

        2. $$\begin{array}{ll} \dfrac{2 + \overline{z}}{2 - \overline{z}} &= \dfrac{2+1 - i \sqrt 3}{2-1 + i \sqrt 3}\\ & = \dfrac{3 - i \sqrt 3}{1 + i \sqrt 3} \\ & = \dfrac{(3 - i \sqrt 3)( 1 - i \sqrt 3)}{(1 + i \sqrt 3)( 1 - i \sqrt 3)} \\ & = \dfrac{3 -3 i \sqrt{3} -i \sqrt{3}+3i^2}{1^2+\left (\sqrt{3}\right )^2} \\ & = \dfrac{3 -4 i \sqrt{3} -3}{4} \\ & = -i \sqrt{3} \\ \end{array}$$
          Ainsi $p= -i \sqrt{3}$
        3. Les points M, N et P sont les points d'affixes respectives 1, $\dfrac{1}{2}\overline{z}$ et $p$. Placer ces trois points dans le repère.
          Justifier l'alignement de ces trois points.

          • L'affixe du vecteur $\vec{MN}$ est $z_{\vec{MN}}=z_N-z_M= \dfrac{1}{2}-i\dfrac{\sqrt 3}{2}-1=-\dfrac{1}{2}-i\dfrac{\sqrt 3}{2}$
          • L'affixe du vecteur $\vec{MP}$ est $z_{\vec{MP}}=z_P-z_M= -i \sqrt{3}-1=-1-i \sqrt 3 $
          • De $z_{\vec{MP}}=2z_{\vec{MN}}$ on déduit l'égalité $\vec{MP}=2 \vec{MN}$ on déduit que les vecteurs $\vec{MN}$ et $\vec{MP}$ sont colinéaires donc les points M, N et P sont alignés.

      PARTIE B

      Soit $u$ le nombre complexe défini par $u = \dfrac{1}{2}z$.

      1. Écrire $u$ sous la forme exponentielle.

        1. Donner l'écriture exponentielle puis l'écriture algébrique de $u^3$.

        2. $u = \dfrac{1}{2}z$ donc $u$ est le nombre complexe de module 1 et d'argument $\dfrac{\pi}{3}$ d'où :$u=e^{i\frac{\pi}{3}}$
          On a alors $u^3= \left (e^{i\frac{\pi}{3}}\right )^3= e^{i\frac{3\pi}{3}}=e^{i\pi}$ $$u^3= e^{i\pi}=-1$$
        3. Vérifier les relations suivantes : $u^4 = - u$ et $u^5 = - u^2$.

        4. $$u^4=u^3 \times u =-1 \times u=-u$$ $$u^5=u^3 \times u^2 =-1 \times u^2=-u^2$$
        5. Vérifier que $1 + u + u^2 + u^3 + u^4 + u^5 + u^6 = 1$.

        6. $$u^6=\left (u^3\right )^2=(-1)^2$$ On a alors : $$1 + u + u^2 + u^3 + u^4 + u^5 + u^6=1 + u + u^2 -1 -u-u^2 + 1=1$$

      Exercice 4 6 points


      Fonction logarithme On note $f$ la fonction définie sur l'intervalle $]0~;~+ \infty[$ par \[f(x) = (2 - \ln x) \ln x.\] Sa courbe représentative $\mathcal{C}_{f}$ dans un repère orthonormal est donnée sur la feuille ANNEXE .

      1. Lire sur le graphique la limite de la fonction f en O. Retrouver ce résultat à l'aide de l'expression de f(x).
      2. Montrer que la fonction dérivée de f sur l'intervalle $]0~;~+ \infty[$ est définie par $f'(x) = \dfrac{2(1 - \ln x)}{x}$.
      3. Étudier le signe de $f'(x)$ lorsque $x$ est dans l'intervalle $]0~;~+ \infty[$ puis donner les variations de la fonction $f$ sur l'intervalle $]0~;~+ \infty[$.
        1. On appelle A et B les points d'intersection de la courbe $\mathcal{C}_{f}$ avec l'axe des abscisses (Voir le graphique). Calculer les abscisses des points A et B.
        2. Calculer le coefficient directeur de la tangente $\mathcal{T}$ à la courbe $\mathcal{C}_{f}$ au point A. Tracer la droite $\mathcal{T}$ sur le graphique donné en annexe.
      4. Montrer que la fonction $F$ définie par \[F(x) = - x(\ln x)^2 + 4x \ln x - 4x\] est une primitive de la fonction $f$ sur l'intervalle $]0~;~+ \infty[$.
      5. On note $\mathcal{D}$ le domaine du plan limité par la courbe $\mathcal{C}_{f}$, l'axe des abscisses et les droites d'équations respectives $x = 1$ et $x = \text{e}^2$.
        1. Hachurer sur le graphique donné en annexe le domaine $\mathcal{D}$.
        2. Calculer l'aire du domaine $\mathcal{D}$.

      Annexe l'exercice 4


      Fonction logarithme

       


      Exercice 4 6 points


      Fonction logarithme On note $f$ la fonction définie sur l'intervalle $]0~;~+ \infty[$ par \[f(x) = (2 - \ln x) \ln x.\] Sa courbe représentative $\mathcal{C}_{f}$ dans un repère orthonormal est donnée sur la feuille ANNEXE .

      1. Lire sur le graphique la limite de la fonction f en O. Retrouver ce résultat à l'aide de l'expression de f(x).

      2. Graphiquement, il semblerait que la courbe $\mathcal{C}_{f}$ admette pour asymptote l'axe des ordonnées. On en déduit, par lecture graphique, que la limite de la fonction $f$ en 0 est $-\infty$
         $\left.\begin{array}{l} \lim\limits_{x \to 0^+}~\ln x=-\infty\\ \lim\limits_{x \to 0^+}~2 - \ln x=+\infty \end{array}\right\}$ par produit on obtient: $\lim\limits_{x \to 0 ^+} (2 - \ln x) \ln x = -\infty$
        $$\lim\limits_{x \to 0 ^+} f(x) = -\infty$$
      3. Montrer que la fonction dérivée de f sur l'intervalle $]0~;~+ \infty[$ est définie par $f'(x) = \dfrac{2(1 - \ln x)}{x}$.

      4. $f$ est dérivable comme produit de deux fonctions dérivables : $f=uv$ d'où $f'=u' v+uv' $ avec pour tout réel $x$, dans $D_{ f}$ :
        $\left\{ \begin{array}{l} u(x)~ =\ln x \\ v(x)~ =2 - \ln x \end{array}\right.$ ainsi : $\left\{ \begin{array}{l} u'(x)~ =\dfrac{1}{x} \\ v'(x)~ =-\dfrac{1}{x} \end{array}\right.$
        Soit pour tout réel $x$ strictement positif, $f$ est dérivable comme produit de deux fonctions dérivables : $f=uv$ d'où $f'=u' v+uv' $ avec pour tout réel $x$, dans $D_{ f}$ :
        $\left\{ \begin{array}{l} u(x)~ =\ln x \\ v(x)~ =2 - \ln x \end{array}\right.$ ainsi : $\left\{ \begin{array}{l} u'(x)~ =\dfrac{1}{x} \\ v'(x)~ =-\dfrac{1}{x} \end{array}\right.$
        Ainsi pour tout réel $x$, dans $]0~;~+ \infty[$ : $$\begin{array}{ll} f'(x) &= \dfrac{1}{x}\times \left (2 - \ln x\right ) +\left (-\dfrac{1}{x}\right ) \times \ln x \\ & = \dfrac{ 2-\ln x - \ln x}{x} \\ & = \dfrac{ 2-2 \ln x }{x} \\ & = \dfrac{ 2\left ( 1 - \ln x \right ) }{x} \\ \end{array}$$
        La dérivée de la fonction $f $ est la fonction $f'$ définie sur l'intervalle $]0~;~+ \infty[$ par $f'(x)= \dfrac{ 2\left ( 1 - \ln x \right ) }{x}$
      5. Étudier le signe de $f'(x)$ lorsque $x$ est dans l'intervalle $]0~;~+ \infty[$ puis donner les variations de la fonction $f$ sur l'intervalle $]0~;~+ \infty[$.

      6. Sur l'intervalle $]0~;~+ \infty[$, $f'(x)$ est du même signe que $1-\ln x $ et, pour tout réel $x$ strictement positif,
        $$\begin{array}{lll} \\ f'(x) > 0 & \iff 1-\ln x > 0 & \\ & \iff -\ln x > -1 & \text{ en ajoutant } -1\\ & \iff \ln x < 1 & \text{ en multipliant par } -1 < 0 \\ & \iff e^{ \ln x } < e^1 & \text{ en appliquant la fonction exponentielle } \\ & \iff 0 < x < e & \text{ strictement croissante sur } ]0~;~+ \infty[\\ \end{array}$$
        Les variations de la fonction $f$ se déduisent du signe de sa dérivée :
        Antilles 2014 TSTI2D Ex4 tab var
        $f(e)= \left (2-\ln(e )\right )\ln e=1 $
        $\left.\begin{array}{l} \lim\limits_{x \to +\infty}~ \ln x=+\infty \\ \lim\limits_{x \to +\infty}~2- \ln x=-\infty \end{array}\right\}$ par produit on obtient:
        $\lim\limits_{x \to +\infty}f(x)=-\infty$
        1. On appelle A et B les points d'intersection de la courbe $\mathcal{C}_{f}$ avec l'axe des abscisses (Voir le graphique). Calculer les abscisses des points A et B.

        2. Les abscisses des points A et B sont les réels $x$ strictement positifs, solutions de l'équation $f(x)=0$. Soit les réels $x$ strictement positifs tels que : $$\begin{array}{lll} \\ f(x) = 0 & \iff (2 - \ln x) \ln x = 0 \\ & \iff -\ln x > (2 - \ln x) = 0 \text{ ou } \ln x =0\\ & \iff \ln x =2 \text{ ou } \ln x =0 \\ & \iff x =e^2 \text{ ou } x =e^0 \\ & \iff x =e^2 \text{ ou } x =1 \\ \end{array}$$
          Les coordonnées des points A et B sont : A$ (1, 0)$ et B$(e^2;0)$.
        3. Calculer le coefficient directeur de la tangente $\mathcal{T}$ à la courbe $\mathcal{C}_{f}$ au point A. Tracer la droite $\mathcal{T}$ sur le graphique donné en annexe.

        4. Le coefficient directeur de la tangente $\mathcal{T}$ à la courbe $\mathcal{C}_{f}$ au point A d'abscisse 1 est $f'(1)=2$.

      7. Montrer que la fonction $F$ définie par \[F(x) = - x(\ln x)^2 + 4x \ln x - 4x\] est une primitive de la fonction $f$ sur l'intervalle $]0~;~+ \infty[$.

      8. Pour tout réel $x$ strictement positif, $F(x) =x\left ( (-\ln x)^2 + 4 \ln x - 4\right )$.
        $F$ est dérivable comme produit de deux fonctions dérivables : $f=uv$ d'où $f'=u' v+uv' $ avec pour tout réel $x$, dans $]0~;~+ \infty[$ :
        $\left\{ \begin{array}{l} u(x)~ =x \\ v(x)~ =(-\ln x)^2 + 4 \ln x - 4 \end{array}\right.$ ainsi : $\left\{ \begin{array}{l} u'(x)~ =1 \\ v'(x)~ =-2 \times \dfrac{1}{x} \times \ln x+ \dfrac{4}{x} =\dfrac{4 - 2 \ln x}{x}\end{array}\right.$
        pour tout réel $x$, dans $]0~;~+ \infty[$ :
        $\begin{array}{ll} F'(x) &= 1\times \left (-(\ln x)^2 + 4 \ln x - 4\right ) +\dfrac{4 - 2 \ln x}{x} \times x \\ & = -(\ln x)^2 + 4 \ln x - 4 +4 - 2 \ln x \\ & = -(\ln x)^2 + 2 \ln x \\ & = (2 - \ln x) \ln x \\ \end{array}$
        Pour tout réel $x$ strictement positif, $F′(x)=f(x)$ donc la fonction $F$ définie par $F(x) = - x(\ln x)^2 + 4x \ln x - 4x$ est une primitive de la fonction $f$ sur l'intervalle $]0~;~+ \infty[$.
      9. On note $\mathcal{D}$ le domaine du plan limité par la courbe $\mathcal{C}_{f}$, l'axe des abscisses et les droites d'équations respectives $x = 1$ et $x = \text{e}^2$.
        1. Hachurer sur le graphique donné en annexe le domaine $\mathcal{D}$.

        2. $\mathcal{D}$ est le domaine sous la courbe $\mathcal{C}_{f}$ colorié.

        3. Calculer l'aire du domaine $\mathcal{D}$.

        4. $$\begin{array}{ll} \mathcal{A}&= \displaystyle\int_{1}^{ \text{e}^2}f(x)\:\text{d}x&\\ & = \left [ F(x)\right ]_{1}^{ \text{e}^2} &\\ & = \left [ - x(\ln x)^2 + 4x \ln x - 4x\right ]_{1}^{ \text{e}^2} &\\ & = - \text{e}^2\left (\ln \left(\text{e}^2\right)\right )^2 + 4\text{e}^2 \ln \left (\text{e}^2\right ) - 4 \text{e}^2 -(-4)& \text{Or } \ln \left (\text{e}^2\right )=2 \ln \text{e} =2 \\ & =-4 &\\ \end{array}$$
          L'aire du domaine $\mathcal{D}$ est égale à 4 unités d'aire.
      • Vues: 10544

      Baccalauréat STI2D Métropole 19 juin 2014

      Exercice 1 4 points


      Probabilités

      Une chocolaterie industrielle fabrique des tablettes de chocolat de 200 grammes. Une machine qui fabrique les tablettes est préréglée afin de respecter cette masse de 200 grammes.
      Lors de la fabrication, toutes les tablettes de chocolat sont pesées et celles dont la masse est inférieure à 195 grammes sont rejetées. L'entreprise ne les commercialisera pas sous cette forme.

      On désigne par X la variable aléatoire qui, à une tablette de chocolat prélevée au hasard dans la production, associe sa masse en grammes.
      On admet que X suit la loi normale d'espérance 200 et d'écart type 2,86.Les résultats seront arrondis à $ 10^{-4}$.

        1. Déterminer la probabilité de l'événement «  195 <Z< 205 » .
        2. Déterminer la probabilité qu'une tablette de chocolat prise au hasard dans la production ne soit pas rejetée après pesée.
      1. Une étude statistique a établi que, si la machine est bien réglée, la proportion de tablettes de chocolat rejetées est de 4 %. Afin de vérifier le réglage de la machine, le responsable qualité prélève de manière aléatoire un échantillon de 150 tablettes et observe que 10 tablettes sont rejetées.
        Cette observation remet-elle en cause le réglage de la machine ? (On pourra utiliser un intervalle de fluctuation.)

      Correction de l'exercice 1 (5 points)


      Probabilités

      Une chocolaterie industrielle fabrique des tablettes de chocolat de 200 grammes. Une machine qui fabrique les tablettes est préréglée afin de respecter cette masse de 200 grammes.
      Lors de la fabrication, toutes les tablettes de chocolat sont pesées et celles dont la masse est inférieure à 195 grammes sont rejetées. L'entreprise ne les commercialisera pas sous cette forme.

      On désigne par X la variable aléatoire qui, à une tablette de chocolat prélevée au hasard dans la production, associe sa masse en grammes.
      On admet que X suit la loi normale d'espérance 200 et d'écart type 2,86.Les résultats seront arrondis à $ 10^{-4}$.

            1. Déterminer la probabilité de l'événement «  195 <Z< 205 » .
          2ND   DISTR   NORMALCDF( 195  , 205,200 
                
          ,2.86)EXE 

              $Normalcdf(195,205,200,2.86))\approx 0,9196$


          $P(195 <Z< 205)\approx 0,9196$ à $ 10^{-4}$ près.
            1. Déterminer la probabilité qu'une tablette de chocolat prise au hasard dans la production ne soit pas rejetée après pesée.
            2. On veut calculer $P(Z\geq 195) =NormalFrép(195,10 ^{99} ,200,2.86)\approx 0,9598$

          $P (Z\geq 195)\approx 0,9598 à 10^{-4}$ près.
        1. Une étude statistique a établi que, si la machine est bien réglée, la proportion de tablettes de chocolat rejetées est de 4 %. Afin de vérifier le réglage de la machine, le responsable qualité prélève de manière aléatoire un échantillon de 150 tablettes et observe que 10 tablettes sont rejetées.
          Cette observation remet-elle en cause le réglage de la machine ? (On pourra utiliser un intervalle de fluctuation.)
        2. Remarque : l'énoncé ne sous-entend pas que la taille de la production est supposée suffisamment grande pour que ce prélèvement puisse être assimilé à  1000 tirages successifs avec remise et donc le tirage de l'échantillon puisse se faire dans les conditions d'application d'une loi binomiale... il faudrait toujours le préciser.

       

          Ici $ p = 0,04, n = 1 50$,donc on a bien $n \geq 30, np =6 \geq 5, n(1- p)=144 \geq 5$ ; ainsi les conditions d'utilisation de l'intervalle de fluctuation asymptotyque sont respectées. L'intervalle $\tilde{I_n}=\left [p-1,96\sqrt{\dfrac{pq}{n}};p+1,96\sqrt{\dfrac{pq}{n}}\right ]=\left [0,04-1,96\sqrt{\dfrac{0,04\times 0,96}{150}}; 0,04+1,96\sqrt{\dfrac{0,04\times 0,96}{150}}\right ]\approx [0,0086 ; 0,0714]$ de fluctuation asymptotique au seuil de 0,95 est donc environ $ [0,0086 ; 0,0714]$.

       

        Sur 150 tablettes, 10 sont non conformes, soit une fréquence de $\dfrac{10}{150}\approx 0,0667$ : cette fréquence appartient  à l'intervalle de fluctuation de fluctuation asymptotique au seuil de 95 %. La machine est bien réglée.

      Exercice 2 4 points


      QCM nombres complexes

      Cet exercice est un questionnaire a choix multiples. Pour chacune des questions suivantes, une seule des quatre réponses proposées est exacte. Aucune justification n'est demandée. Une bonne réponse rapporte un point. Une mauvaise réponse, plusieurs réponses ou l'absence de réponse à une question ne rapportent ni n'enlèvent de point. Indiquer sur la copie le numéro de la question et la réponse correspondante.
      On considère les deux nombres complexes $z=2e^{i\frac{2\pi}{3}}$ et $z' =2e^{-i\frac{2\pi}{3}}$

      1. La forme algébrique de $z$ est égale à :
        1. $z = -l + i \sqrt 3$
        2. $z = l + i \sqrt 3$
        3. $z = 2 + i \sqrt 3$
        4. $z = \sqrt 3 -i$
      2. Le nombre complexe $z'$ est le nombre complexe :
        1. opposé de $z$
        2. inverse de $z$
        3. conjugué de $z$
        4. opposé du conjugué de $z$
      3. Le nombre complexe $z \times z’$:
        1. est un nombre réel
        2. est un nombre imaginaire pur
        3. a pour module 2
        4. est un nombre complexe dont un argument est $\dfrac{4\pi}{3}$
      4. Un argument du nombre complexe $z"$ tel que $z \times z" = i $ est :
        1. $\dfrac{\pi}{3}$
        2. $\dfrac{5\pi}{6}$
        3. $\dfrac{\pi}{6}$
        4. $-\dfrac{\pi}{6}$

      Correction de l'exercice 2 (5 points)

      Cet exercice est un questionnaire a choix multiples. Pour chacune des questions suivantes, une seule des quatre réponses proposées est exacte. Aucune justification n'est demandée. Une bonne réponse rapporte un point. Une mauvaise réponse, plusieurs réponses ou l'absence de réponse à une question ne rapportent ni n'enlèvent de point. Indiquer sur la copie le numéro de la question et la réponse correspondante.
      On considère les deux nombres complexes $z=2e^{i\frac{\pi}{3}}$ et $z' =2e^{-i\frac{\pi}{3}}$
      1. La forme algébrique de $z$ est égale à :
        $z=2e^{i\frac{2\pi}{3}}=2\left(\cos\left(\frac{2\pi}{3}\right)+i\sin\left(2\frac{\pi}{3}\right)\right)=2\left(-\dfrac{1}{2}+i\dfrac{\sqrt 3}{2}\right)= -1 + i \sqrt 3$
        1. $z =-1 + i \sqrt 3$ VRAI
      • Le nombre complexe $z'$ est le nombre complexe :
        1. conjugué de $z$ car $\overline{ re^{i\theta}} =re^{-i\theta}$ VRAI
      • Le nombre complexe $z \times z '= 2e^{i\frac{2\pi}{3}}\times 2e^{-i\frac{2\pi}{3}} = 4 e^{i\frac{2\pi}{3}-i\frac{2\pi}{3}}=4e^0=4$:
        1. est un nombre réel VRAI
      • Un argument du nombre complexe $z"$ tel que $z \times z" = i $ est :
        En prenant les arguments de part et d'aure, il vient : $$arg(z \times z") =arg( i) \;(1)$$ $$arg(z )+ arg(z") =\dfrac{\pi}{2} $$ $$ \text{ Or } z=2e^{i\frac{2\pi}{3}} \text{ d'où on déduit } arg(z)=\dfrac{2\pi}{3} $$ $$(1)\iff \dfrac{2\pi}{3} + arg(z") =\dfrac{\pi}{2} \iff arg(z") = \dfrac{\pi}{2} -\dfrac{2\pi}{3} =- \dfrac{\pi}{6}$$
        1. $-\dfrac{\pi}{6}$ VRAI

       


      Exercice 3 6 points


      Equations différentielles

      Dans cet exercice, la température est exprimée en degrés Celsius ( °C) et le temps t est exprimé en heures. Une entreprise congèle des ailerons de poulet dans un tunnel de congélation avant de les conditionner en sachets. A l'instant $t=0$, les ailerons, à une température de 5 °C, sont placés dans le tunnel. Pour pouvoir respecter la chaîne du froid, le cahier des charges impose que les ailerons aient une température inférieure ou égale à -24 °C.

      Partie A

      La température des ailerons dans le tunnel de congélation est modélisêe en fonction du temps $t$ par la fonction définie sur l'intervalle $[0,+\infty[$ par $f(t) =35e^{-1,6t}-30$.

      1. Déterminer la température atteinte par les ailerons au bout de 30 minutes, soit 0,5 h.
      2. Étudier le sens de variation de la fonction $f$.
      3. Si les ailerons de poulet sont laissés une heure et demie dans le tunnel de congélation, la température des ailerons sera-t-elle conforme au cahier des charges ?
      4. Résoudre par le calcul l’équation $f(t)=-24$ et interpréter le résultat trouvé.

      Partie B

      Pour moderniser son matériel, l'entreprise a investi dans un nouveau tunnel de congélation. La,température des ailerons dans ce nouveau tunnel est modélisêe, en fonction du temps, par une fonction $g$ de'finie et derivable sur l'intervalle $[0,+\infty[$, qui est solution de l'équation différentielle $y' + l,5y = -52,5$.

      1. Résoudre l'équation différentielle $y' + l,5y = -52,5$.
        1. Justifier que $g(0) = 5$.
        2. Vérifier que la fonction $g$ est définie par $ g(t)=40e^{-1,5t}-35$
      2. Ce nouveau tunnel permet-il une congélation plus rapide ?

       


      Correction de l'exercice 3 (5 points)


      Equations différentielles

      Dans cet exercice, la température est exprimée en degrés Celsius ( °C) et le temps t est exprimé en heures. Une entreprise congèle des ailerons de poulet dans un tunnel de congélation avant de les conditionner en sachets. A l'instant $t=0$, les ailerons, à une température de 5 °C, sont placés dans le tunnel. Pour pouvoir respecter la chaîne du froid, le cahier des charges impose que les ailerons aient une température inférieure ou égale à -24 °C.

      Partie A

      La température des ailerons dans le tunnel de congélation est modélisêe en fonction du temps $t$ par la fonction définie sur l'intervalle $[0,+\infty[$ par $f(t) =35e^{-1,6t}-30$.

        1. Déterminer la température atteinte par les ailerons au bout de 30 minutes, soit 0,5 h.
        2. On calcule $f(0,5)=35e^{-1,6 \times 0,5}-30=35e^{-0,8}-30\approx -14,3$


      La température atteinte par les ailerons au bout de 30 minutes sera d'environ -14 °C

        1. Étudier le sens de variation de la fonction $f$.
        2. On calcule la dérivée et on étudie son signe : $$f'(t)=35\times (-1,6)e^{-1,6t} = -56e^{-1,6t}$$ On a utilisé la formule de dérivation : $$\left (e^u\right )'=u'e^u$$ Signe de la dérivée : La fonction exponentielle étant strictement positive sur $\mathbb{R}$ on déduit que pour tout $t\in [0;+\infty[$, on a $f'(t)<0$


      La fonction $f$ est strictement décroissante sur $[0;+\infty[$.

        1. Si les ailerons de poulet sont laissés une heure et demie dans le tunnel de congélation, la température des ailerons sera-t-elle conforme au cahier des charges ?
        2. $f(1,5)\approx -26,8 °C$

      La température des ailerons sera conforme au cahier des charges car inférieure ou égale à -24 °C

        1. Résoudre par le calcul l’équation $f(t)=-24$ et interpréter le résultat trouvé.
        2. $$\begin{array} {l l l} f(t)=-24& \iff 35e^{-1,6t}-30=-24 & \\ & \iff 35e^{-1,6t}=6 &\\ & \iff e^{-1,6t} =\dfrac{6}{35}&\\ & \iff -1,6t =\ln\left(\dfrac{6}{35}\right)&\text{ en appliquant la fonction } \ln\\ & \iff t = \dfrac{ \ln\left(\dfrac{6}{35}\right)}{-1,6} & \\ & \iff t \approx 1,10 h &\\ \end{array}$$

      La température des ailerons atteindra -24 °C et sera donc conforme au cahier des charges au bout de 1h et 6 minutes et 9 secxondes.

      Partie B

      Pour moderniser son matériel, l'entreprise a investi dans un nouveau tunnel de congélation. La,température des ailerons dans ce nouveau tunnel est modélisêe, en fonction du temps, par une fonction $g$ de'finie et derivable sur l'intervalle $[0,+\infty[$, qui est solution de l'équation différentielle $y' + 1,5y = -52,5$.

        1. Résoudre l'équation différentielle $y' + l,5y = -52,5$.
        2. L'équation différentielle $y' + l,5y = -52,5$ se met sous la forme $y' =1,5y -52,5$.

       

          Elle est du type $y' =a y +b$ où $a=-1,5$ et $b=-52,5$, les solutions de cette équation sont donc les fonctions définies sur $\mathbb{R}$ par $g(t)=C e^{a t} -\dfrac{b}{a}$ , soit ici

       

          $g(t)=C e^{-1,5 t} -35$ où $C$ désigne une constante réelle quelconque.


      Les solutions de cette équation sont donc les fonctions définies sur $\mathbb{R}$ par $g(t)=C e^{-1,5 t} -35$ où $C$ désigne une constante réelle quelconque.

            1. Justifier que $g(0) = 5$.
            2. $g(0) = 5$ car l'instant $t=0$, les ailerons, sont à une température de 5 °C.
            3. Vérifier que la fonction $g$ est définie par $ g(t)=40e^{-1,5t}-35$
            4. $$g(0)=5\iff C e^{-1,5 \times 0} -35 =5 $$ $$g(0)=5\iff C e^0 -35 =5 $$ $$C=40$$
          La fonction $g$ est définie par $ g(t)=40e^{-1,5t}-35$
        1. Ce nouveau tunnel permet-il une congélation plus rapide ?
        2. On résout $g(t)=-24$ $$\begin{array} {l l l} g(t)=-24& \iff 40e^{-1,5t}-35=-24 & \\ & \iff 40 e^{-1,5t}=11 &\\ & \iff e^{-1,5t} =\dfrac{11}{40}&\\ & \iff -1,5t =\ln\left(\dfrac{11}{40}\right)&\text{ en appliquant la fonction } \ln\\ & \iff t = \dfrac{ \ln\left(\dfrac{11}{40}\right)}{-1,5} & \\ & \iff t \approx 0,86 h \approx 52 \, min &\\ \end{array}$$

      Ce nouveau tunnel permet  une congélation plus rapide.


      Exercice 4 6 points


      Suites

       

      Au cours de son évolution, une tornade se déplace dans un corridor de quelques centaines de mètres de large sur quelques kilomètres de long.

      Document 1 :

      L'échelle de Fujita est une échelle servant à classer les tornades par ordre de gravité, en fonction des dégâts qu'elles occasionnent. Une partie de cette échelle est présentée dans le tableau ci-dessous.

      Catégorie Vitesse des vents en km.h${-1}$ Dégâts occasionnés
      F0 60 à 120 Dégâts légers : dégâts sur cheminées, arbres, fenêtres,...
      F1 120 à 180 Dégâts modérés : automobiles renversées, arbres déracinés,...
      F2 180 à 250 Dégâts importants : toits arrachés, hangars et dépendances démolis,...
      F3 250 à 330 Dégâts considérables : murs extérieurs et toits projetés, maisons et bâtiments de métal effondrés, forêts abattues,...
      F4 330 à 420 Dégâts dévastateurs : murs effondrés, objets en acier ou en béton projetés comme des missiles,...
      F5 420 à 510 Dégâts incroyables : maisons rasées ou projetées sur de grandes distances, murs extérieurs et toits arrachés sur de gros bâtiments,...

      Document 2 :

      A partir des mesures relevées lors d'observations de phénomènes semblables, des météorologues ont admis ia règle suivante :«  la vitesse des vents dans les tornades diminue régulièrement de 10 % toutes les5 minutes » .

      On appelle «  durée de vie »  d'une tornade le temps nécessaire, depuis sa formation, pour que la vitesse des vents devienne inférieure a 120 km.h $^{-1}$

       

      Lors de la formation d'une tornade, on a mesuré la vitesse des vents par un radar météorologique et on a trouvé une vitesse initiale de 420 km.h $^{-1}$.

      L'objectif de ce problème est d'estimer la durée de vie de cette tornade.

      Dans cet exercice, les résultats seront arrondis à 10 km.h $^{-1}$ .

        1. Cinq minutes après la mesure initiale, la vitesse des vents est de 378 km.h $^{-1}$. Vérifier que ce résultat correspond à la régie admise.
          A quelle catégorie appartient la tornade à ce moment là?
        2. Vérifier que, quinze minutes après la mesure initiale, cette tornade occasionne des dégâts classés comme «  dégâts considérables » .
      1. Pour déterminer la durée de vie de cette tornade, un étudiant propose de modéliser le phénomène par une suite géométrique de raison q. Il commence à élaborer l'algorithme ci-dessous. $$ \begin{array}{|l |l |}\hline \text{ Variables :} & \\ & n : \text{ un nombre entier naturel }\\ & v : \text{ un nombre réel } \\ &q : \text{ un nombre réel } \\ \text{ Initialisation :}&\\ &\text{ Affecter à } n \text{ la valeur 0 }\\ &\text{ Affecter à } v \text{ la valeur 420 }\\ &\text{ Affecter à } q \text{ la valeur 0,9 }\\ \text{ Traitement :}& \\ &\text{ Tant que } \ldots \\ & \ldots\ldots\ldots\\ & \ldots\ldots\ldots \\ &\text{ Fin Tant que }\\ \text{ Sortie : }& \\ & \text{ Afficher } 5\times n \\\hline \end{array}$$
        1. Justifier la valeur 0,9 dans la phrase «  Affecter à q la valeur 0,9 » .
        2. Donner le premier terme et la raison de la suite géométrique proposée par l'étudiant.
        3. Dans l'algorithme ci-dessus, des pointillés indiquent des parties manquantes.
          Recopier la partie relative au traitement et la compléter pour que l'étudiant puisse déterminer la durée de vie de cette tornade.
        4. Expliquer l'instruction «  Afficher 5 $\times n $ » proposée par l'étudiant.
      2. On désigne par $(v_n)$ la suite géométrique proposée par l'étudiant. Exprimer $v_n$ en fonction de $n$.
      3. Déterminer la durée de vie de cette tornade au sens défini dans le document 2.

       


      Correction de l'exercice 4 6 points


      Suites

      Au cours de son évolution, une tornade se déplace dans un corridor de quelques centaines de mètres de large sur quelques kilomètres de long.

      Document 1 :

      L'échelle de Fujita est une échelle servant à classer les tornades par ordre de gravité, en fonction des dégâts qu'elles occasionnent. Une partie de cette échelle est présentée dans le tableau ci-dessous.

      Catégorie Vitesse des vents en km.h${-1}$ Dégâts occasionnés
      F0 60 à 120 Dégâts légers : dégâts sur cheminées, arbres, fenêtres,...
      F1 120 à 180 Dégâts modérés : automobiles renversées, arbres déracinés,...
      F2 180 à 250 Dégâts importants : toits arrachés, hangars et dépendances démolis,...
      F3 250 à 330 Dégâts considérables : murs extérieurs et toits projetés, maisons et bâtiments de métal effondrés, forêts abattues,...
      F4 330 à 420 Dégâts dévastateurs : murs effondrés, objets en acier ou en béton projetés comme des missiles,...
      F5 420 à 510 Dégâts incroyables : maisons rasées ou projetées sur de grandes distances, murs extérieurs et toits arrachés sur de gros bâtiments,...

      Document 2 :

      A partir des mesures relevées lors d'observations de phénomènes semblables, des météorologues ont admis ia règle suivante :«  la vitesse des vents dans les tornades diminue régulièrement de 10 % toutes les5 minutes » .

      On appelle «  durée de vie »  d'une tornade le temps nécessaire, depuis sa formation, pour que la vitesse des vents devienne inférieure a 120 km.h $^{-1}$

       

      Lors de la formation d'une tornade, on a mesuré la vitesse des vents par un radar météorologique et on a trouvé une vitesse initiale de 420 km.h $^{-1}$.

      L'objectif de ce problème est d'estimer la durée de vie de cette tornade.

      Dans cet exercice, les résultats seront arrondis à 10 km.h $^{-1}$ .

            1. Cinq minutes après la mesure initiale, la vitesse des vents est de 378 km.h $^{-1}$. Vérifier que ce résultat correspond à la régle admise.
              A quelle catégorie appartient la tornade à ce moment là?
            2. $$v=420 -10\% =0,9\times 420 = 378$$
          Ainsi cinq minutes après la mesure initiale, la vitesse des vents est de 378 km.h $^{-1}$ et donc la tornade est de catégorie F4.
            1. Vérifier que, quinze minutes après la mesure initiale, cette tornade occasionne des dégâts classés comme «  dégâts considérables » .
              • Après 5 minutes la vitesse de la tornade est $v_1=0,9 \times v_0=0,9\times 420 = 378$
              • Après 10 minutes la vitesse de la tornade est $v_2=0,9 \times v_1=0,9\times 378 \approx 340$
              • Après 15 minutes la vitesse de la tornade est $v_3=0,9 \times v_2\approx0,9\times 340 \approx 306$
          Ainsi quinze minutes après la mesure initiale, la vitesse des vents est environ de 306 km.h $^{-1}$ et donc la tornade est de catégorie F3 et occasionne des «  dégâts considérables ».
        1. Pour déterminer la durée de vie de cette tornade, un étudiant propose de modéliser le phénomène par une suite géométrique de raison q. Il commence à élaborer l'algorithme ci-dessous. $$ \begin{array}{|l |l |}\hline \text{ Variables :} & \\ & n : \text{ un nombre entier naturel }\\ & v : \text{ un nombre réel } \\ &q : \text{ un nombre réel } \\ \text{ Initialisation :}&\\ &\text{ Affecter à } n \text{ la valeur 0 }\\ &\text{ Affecter à } v \text{ la valeur 420 }\\ &\text{ Affecter à } q \text{ la valeur 0,9 }\\ \text{ Traitement :}& \\ &\text{ Tant que } \ldots \\ & \ldots\ldots\ldots\\ & \ldots\ldots\ldots \\ &\text{ Fin Tant que }\\ \text{ Sortie : }& \\ & \text{ Afficher } 5\times n \\\hline \end{array}$$
            1. Justifier la valeur 0,9 dans la phrase «  Affecter à q la valeur 0,9 » .
            2. D'après l'ennoncé des météorologues ont admis la règle suivante :«  la vitesse des vents dans les tornades diminue régulièrement de 10 % toutes les5 minutes »  .

              Comme diminuer une grandeur de t % équivaut à multiplier sa valeur par $1- \dfrac{t}{100} $, ici pour diminuer la vitesse de 10 % on multiplie sa valeur par $1- \dfrac{10}{100}=0,9 $
            1. Donner le premier terme et la raison de la suite géométrique proposée par l'étudiant.
            2. Le premier terme est la valeur initiale de $v$ soit $v_0=420$, sa raison est $q=0,9$.
            3. Dans l'algorithme ci-dessus, des pointillés indiquent des parties manquantes.
              Recopier la partie relative au traitement et la compléter pour que l'étudiant puisse déterminer la durée de vie de cette tornade.
            4. $$ \begin{array}{|l |l |} \hline \text{ Variables :} & \\ & n : \text{ un nombre entier naturel }\\ & v : \text{ un nombre réel } \\ &q : \text{ un nombre réel } \\ \text{ Initialisation :}&\\ &\text{ Affecter à } n \text{ la valeur 0 }\\ &\text{ Affecter à } v \text{ la valeur 420 }\\ &\text{ Affecter à } q \text{ la valeur 0,9 }\\ \text{ Traitement :}& \\ &\text{ Tant que } v < 120 \\ & \text{ Affecter à } v \text{ la valeur }q \times v \\ & \text{ Affecter à } n \text{ la valeur } n+1 \\ &\text{ Fin Tant que }\\ \text{ Sortie : }& \\ & \text{ Afficher } 5\times n \\ \hline \end{array}$$
            5. Expliquer l'instruction Afficher 5 $\times n $ » proposée par l'étudiant.
            6. «  la vitesse des vents dans les tornades diminue régulièrement de 10 % toutes les5 minutes » , ainsi l'instruction «  Afficher 5 $\times n $ » proposée par l'étudiant donne le temps nécessaire, depuis sa formation, pour que la vitesse des vents devienne inférieure a 120 km.h $^{-1}$

          l'instruction «  Afficher 5 $\times n $ » doone la durée de vie en minutes de la tornade.
        2. Déterminer la durée de vie de cette tornade au sens défini dans le document 2.
        3. Notons $v_n$ la vitesse de la tornade après 5 $\times n $, la suite $(v_n)$ est géométrique, donc $v_n=q^n\times v_0=420 \times 0,9^n$

       

          On résout $v_n<120$ $$\begin{array} {l l l} v_n < 120 & \iff 420 \times 0,9^n < 120 & \\ & \iff 0,9^n < \dfrac{120}{420} &\\ & \iff 0,9^n < \dfrac{2}{7} & \\ & \iff ln\left( 0,9^n\right) < \ln\left(\dfrac{2}{7}\right) & \text{ en appliquant la fonction } \ln \text{ strictement croissante sur } ]0;+\infty[ \\ & \iff n \ln\left( 0,9 \right) < \ln\left(\dfrac{2}{7}\right) & \\ & \iff n > \dfrac{\ln\left(\dfrac{2}{7}\right)}{\ln\left( 0,9 \right)} &\text{ en divisant par } \ln\left( 0,9 \right)< 0 \\ \end{array}$$ Or $\dfrac{\ln\left(\dfrac{2}{7}\right)}{\ln\left( 0,9 \right)}\approx 11,89$ , donc $n\geq 12$


      La durée de vie de la tornade est de $5\times 12 =60$ min soit une heure !

      • Vues: 12416

      Rechercher